ChaseDream

标题: 【每日逻辑练习第二季】【1-3】 [打印本页]

作者: bananazoo    时间: 2011-11-22 00:46
标题: 【每日逻辑练习第二季】【1-3】
恩~很高兴看到这么多人参与进来~
真,美好~不是么~
大家加油!

跟帖做题方法请见:
http://forum.chasedream.com/GMAT_CR/thread-613902-1-1.html


【精练】
3. There is relatively little room for growth in the
overall carpet market, which is tied to the size of the
population. Most who purchase carpet do so only
once or twice, first in their twenties or thirties, and
then perhaps again in their fifties or sixties. Thus as
the population ages, companies producing carpet
will be able to gain market share in the carpet market
only through purchasing competitors, and not
through more aggressive marketing.
Which one of the following, if true, casts the most
doubt on the conclusion above?
--BIBLE WEAKEN NO.3
(A) Most of the major carpet producers market
other floor coverings as well.
(B) Most established carpet producers market
several different brand names and varieties,
and there is no remaining niche in the
market for new brands to fill.
(C) Two of the three mergers in the industry’s last
ten years led to a decline in profits and
revenues for the newly merged companies.
(D) Price reductions, achieved by cost-cutting in
production, by some of the dominant firms
in the carpet market are causing other
producers to leave the market altogether.
(E) The carpet market is unlike most markets in
that consumers are becoming increasingly
resistant to new patterns and styles.

【逻辑链】
2.
Scientists are discussing ways to remove excess carbon dioxide from the atmosphere by increasing the amount that is absorbed by plant life.  One plan to accomplish this is to establish giant floating seaweed farms in the oceans.  When the seaweed plants die, they will be disposed of by being burned for fuel.

Which of the following, if true, would indicate the most serious weakness in the plan above?
-- PREP07 CR1 NO.9


(A) Some areas of ocean in the Southern Hemisphere do not contain sufficient nutrients to support large seaweed farms.
(B) When a seaweed plant is burned, it releases an amount of carbon dioxide comparable to the amount it has absorbed in its lifetime.
(C) Even if seaweed farms prove effective, some people will be reluctant to switch to this new fuel.
(D) Each year about seven billion tons of carbon dioxide are released into the atmosphere but only about five billion tons are absorbed by plant life.
(E) Seaweed farms would make more money by farming seaweed to sell as nutritional supplements than by farming seaweed to sell as fuel.


3.
Jennifer:  Video rental outlets in Centerville together handled 10,000 fewer video rentals in 1994 than in 1993.  The decline in rentals was probably due almost entirely to the February 1994 opening of Videorama, the first and only video rental outlet in the area that, in addition to renting videos, also sold them cheaply.


Brad:  There must be another explanation:  as you yourself said, the decline was on the order of 10,000 rentals.  Yet Videorama sold only 4,000 videos in 1994.


Which of the following, if true, would most seriously weaken the force of the objection that Brad presents to Jennifer's explanation?
-- PREP07 CR1 NO.10

(A) In 1994 Videorama rented out more videos than it sold.
(B) In 1994 two new outlets that rent but that do not sell videos opened in Centerville.
(C) Most of the video rental outlets in Centerville rent videos at a discount on certain nights of the week.
(D) People often buy videos of movies that they have previously seen in a theater.
(E) People who own videos frequently loan them to their friends.


4.
Mourdet Winery:  Danville Winery's new wine was introduced to compete with our most popular wine, which is sold in a distinctive tall, black bottle.  Danville uses a similar bottle.  Thus, it is likely that many customers intending to buy our wine will mistakenly buy theirs instead.


Danville Winery:  Not so.  The two bottles can be readily distinguished:  the label on ours, but not on theirs, is gold colored.


Which of the following, if true, most undermines Danville Winery's response?
-- PREP07 CR1 NO.11

(A) Gold is the background color on the label of many of the wines produced by Danville Winery.
(B) When the bottles are viewed side by side, Danville Winery's bottle is perceptibly taller than Mourdet Winery's.
(C) Danville Winery, unlike Mourdet Winery, displays its wine's label prominently in advertisements.
(D) It is common for occasional purchasers to buy a bottle of wine on the basis of a general impression of the most obvious feature of the bottle.
(E) Many popular wines are sold in bottles of a standard design.


5.
Finding of a survey of Systems magazine subscribers:  Thirty percent of all merchandise orders placed by subscribers in response to advertisements in the magazine last year were placed by subscribers under age thirty-five.


Finding of a survey of advertisers in Systems magazine:  Most of the merchandise orders placed in response to advertisements in Systems last year were placed by people under age thirty-five.


For both of the findings to be accurate, which of the following must be true?
-- PREP07 CR1 NO.12

(A) More subscribers to Systems who have never ordered merchandise in response to advertisements in the magazine are age thirty-five or over than are under age thirty-five.
(B) Among subscribers to Systems, the proportion who are under age thirty-five was considerably lower last year than it is now.
(C) Most merchandise orders placed in response to advertisements in Systems last year were placed by Systems subscribers over age thirty-five.
(D) Last year, the average dollar amount of merchandise orders placed was less for subscribers under age thirty-five than for those age thirty-five or over.
(E) Last year many people who placed orders for merchandise in response to advertisements in Systems were not subscribers to the magazine.

作者: bananazoo    时间: 2011-11-22 00:47
Key: D B E D E  答案在此处,把前面的选中就可以看到了~被设成白色的啦~

【精练题解释】
This is another difficult problem but very typical of the LSAT. As always, the
key to success is to isolate the conclusion, which appears in the last sentence:
“companies producing carpet will be able to gain market share in the carpet
market only through purchasing competitors.” As you should have noted while
reading, the conclusion contains a conditional indicator and is thereby
conditional in nature. The conclusion can be diagrammed as:
     GMS = gain market share in the carpet market
     C = purchasing competitors
               GMS --->C
According to the author, to gain market share in the carpet market a company
must purchase a competitor. Answer choice (C) is often selected by students,
but it does not attack this idea. To attack a conditional statement you must show
that the necessary condition is not actually necessary for the sufficient condition
to occur. Answer choice (C) simply suggests that when companies purchase
their competitors the endeavor is often financially unsuccessful. Essentially,
answer choice (C) fails to prove that purchasing competitors is unnecessary to
gain market share. Answer choice (D), on the other hand, does suggest a way
for companies to gain market share without purchasing competitors, thereby
attacking the conditional statement given in the stimulus. Thus, answer choice
(D) is correct.
Answer choice (A): This answer goes beyond the scope of the argument, which
is limited to the carpet market (and not other floor coverings).
Answer choice (B): This is an Opposite answer that strengthens the argument.
If there are no remaining niches to fill, then there is no way to expand other than
to purchase a competitor.
Answer choice (C): This attractive answer is wrong for two very strong
reasons:
1. A Shell Game is played with the details of the conclusion. The
conclusion is about market share. Answer choice (C) is about a
decline in profits and revenues. The two are not the same, and so the
information in the answer choice does not weaken the conclusion.
2. Even if you assume that market share is the same thing as profits and
revenues, a second Shell Game is played because the answer then
attacks a conclusion that is similar but different than the given
conclusion.
If the conclusion were as follows:
                       C ---> GMS
then answer choice (C) would be correct (again, assuming market
share is the same thing as profits and revenues). But, the above is a
Mistaken Reversal of the conclusion, and so the attack is made on a
statement that uses the same terms as the conclusion but puts them in
a different relationship. This is a great example of the cleverness
displayed by the test makers. Fortunately you can avoid this answer if
you know what to look for when attacking conditional reasoning.
More on this topic in the next section.
One point worth noting is that it is no accident that the most tempting wrong
answer choice appears just before the correct answer. This is a classic LSAT
trick, and one that is very effective because most test takers relax once they find
an answer they feel is attractive. This makes them less likely to closely examine
the answers that follow. Never relax during the LSAT!
Answer choice (D): This is the correct answer. If price reductions drive out
some of the carpet producers, then other producers can take the market share
left behind. This scenario shows that a company can gain market share without
purchasing a competitor, thus attacking the necessary condition in the
conclusion.
Answer choice (E): This Opposite answer strengthens the argument. If the
consumers are resistant to new styles, then one fewer possibility exists if a
company is trying to increase market share. By eliminating this option, the
conclusion is strengthened (if you eliminate an idea that would hurt the
argument, that strengthens the argument because it has fewer “competitors.”
More on this later).

作者: fox0923    时间: 2011-11-22 02:23
精练-------〉45s(第一遍没读懂)-----------weaken
P: The consumers only change their carpets in their 20s/30s and 40s/50s.
C: The carpet company can only gain market share from purchasing its competitors, yet not from the carpet market growth.
Prephrase:
The carpet company's competitor have the price as same as this carpet company.
The exportation of carpet can induce more profit for the carpet company.
The promotion plan of the carpet company can be achieved by replacing old carpet with the new ones in a cheaper price.
Analysis:
(A) Most of the major carpet producers market
other floor coverings as well.
----------------------------------------------------------we don't care if the producer market other floor covering or not, since this is not the main point of the passage.
(B) Most established carpet producers market
several different brand names and varieties,
and there is no remaining niche in the
market for new brands to fill.
----------------------------------------------------------new brand names and varieties cannot conclude the carpet company can gain market share through purchasing competitors.
(C) Two of the three mergers in the industry’s last
ten years led to a decline in profits and
revenues for the newly merged companies.
-----------------------------------------------------------this is one example, but it cannot represent for the whole.
(D) Price reductions, achieved by cost-cutting in
production, by some of the dominant firms
in the carpet market are causing other
producers to leave the market altogether.
-----------------------------------------------------------contender.
(E) The carpet market is unlike most markets in
that consumers are becoming increasingly
resistant to new patterns and styles.
------------------------------------------------------------new patterns and styles are irrelevant,this is to support the conclusion.



分析不足:
reasons:
1. A Shell Game is played with the details of the conclusion. The
conclusion is about market share. Answer choice (C) is about a
decline in profits and revenues. The two are not the same, and so the
information in the answer choice does not weaken the conclusion.

对C的理解稍微还不够深刻,其实是shell game,C不能说明market share的增加是由于purchasing competitors引起的,所以即便是profit/revenue下降等等,都不够充分说明weaken的效果。


其次,对于D为什么是对的理解也不到位,用了POE方法排出,但是不能体会精髓!
Answer choice (D): This is the correct answer. If price reductions drive out
some of the carpet producers, then other producers can take the market share
left behind. This scenario shows that a company can gain market share without
purchasing a competitor,
thus attacking the necessary condition in the
conclusion.





问题???
但是highlight部分我还是没太看明白,D到底是怎么证明carpet company can gain market share without purchasing a competitor的?要我看,最多证明carpet company是不能从purchasing competior 身上获利而已阿~~求解!
作者: 南瓜0729    时间: 2011-11-22 07:24
45s
P:people buy C once or twice in their life.
C:as population ages, the company should purchase competitors rather than use more marketing.
P(weaken)eople prefer to the product whose company has compelling marketing method.
E
A irrelevant  floor covering
B support
C irrelevant 说的profit
D ?
E correct


以为that修饰most markets 还纳闷这句话的 计时器挡住的in。。。。。做的太屎了!


30s
P:scientists wants to reduce the amount of CO2. they plan to establish seaweed plant.
C:when plants die, they can be disposed by being burned by fuel.
p(weaken):fuel burning produce CO2.
B


45s
J:video rental outlets decline in 1994 because Videorama opened.
B:the decline was 10000,but Videorama sold only 4000.
P:?
E


28s
M:new wine will compete our wine, because the bottle looks the same.
D:no. labels are different.
P(weaken D): customers cannot see the little difference.
D


P:30% of orders placed by s were s under 35
P: most of orders placed were placed by people under 35.
P(must be true):many people placed order were not s
E
作者: ajcaesar    时间: 2011-11-22 08:19
第一次跟帖,有错请指正

逻辑链:
购买地毯次数少---->市场饱和
为了市占率只有并购,没有行销

选项分析:
一开始我也选(C),后来看了解释才知道自己题意没抓好
(C)
并购后获利下降.............从逻辑链来看就知道无关
(D)
削价竞争,迫使没资金的竞争对手离场,之后赢得市占率

有错请鞭..............
作者: 小意达de花儿    时间: 2011-11-22 09:25
占座
作者: CHRISTINE2010    时间: 2011-11-22 10:16
报到先~~
作者: fox0923    时间: 2011-11-22 11:37
第一次跟帖,有错请指正

逻辑链:
购买地毯次数少---->市场饱和
为了市占率只有并购,没有行销

选项分析:
一开始我也选(C),后来看了解释才知道自己题意没抓好
(C)
并购后获利下降.............从逻辑链来看就知道无关
(D)
削价竞争,迫使没资金的竞争对手离场,之后赢得市占率

有错请鞭..............
-- by 会员 ajcaesar (2011/11/22 8:19:38)



明白了D选项了,谢谢翻译了,当时想的太多了,一时间没反应过来~
作者: xeyyxzty    时间: 2011-11-22 12:47
1.background:There is relatively little room for growth in the overall carpet market, which is tied to the size of the population.
premise:Most who purchase carpet do so only once or twice, first in their twenties or thirties, and then perhaps again in their fifties or sixties.
conclusion:as the population ages, companies producing carpet will be able to gain market share in the carpet market only through purchasing competitors, and not through more aggressive marketing.
maybe more aggressive marketing will attract consumers who originally want to buy carpets once~(这个理由不成立吧...)
(A) Most of the major carpet producers market other floor coverings as well.
--it's about the carpet market--irrelevant
(B) Most established carpet producers market several different brand names and varieties, and there is no remaining niche in the
market for new brands to fill.
--i don't know~maybe~irrelevant?--support
(C) Two of the three mergers in the industry’s last ten years led to a decline in profits and revenues for the newly merged companies.
--not about competitors-,but still gain market shares-right
(D) Price reductions, achieved by cost-cutting in production, by some of the dominant firms in the carpet market are causing other producers to leave the market altogether.
--competitors--support--right
(E) The carpet market is unlike most markets in that consumers are becoming increasingly resistant to new patterns and styles.

--it has nothing to do with the choice of consumers--irrelevant--support
果然错了...To attack a conditional statement you must show that the necessary condition is not actually necessary for the sufficient condition to occur.
purchasing competitors...真的只是purchasing competitors~没敢这么想,觉得另有深意
好伤心的一道题~~~2.这个是set of facts 吧~
increase the amount of carbon dioxide absorbed by plant life -->remove excess carbon dioxide from the atmosphere
the plants are burned for fuel--but when they are burned, they make carbon oxide again
(A) Some areas of ocean in the Southern Hemisphere do not contain sufficient nutrients to support large seaweed farms.
--to weaken the plan is to find the reason that the seaweed plants cannot remove the carbon oxide from the atmosphere--irrelevant
(B) When a seaweed plant is burned, it releases an amount of carbon dioxide comparable to the amount it has absorbed in its lifetime.

--right
(C) Even if seaweed farms prove effective, some people will be reluctant to switch to this new fuel.

--the plant works, and some people will use the new fuel--support
(D) Each year about seven billion tons of carbon dioxide are released into the atmosphere but only about five billion tons are absorbed by plant life.

--irrelevant
(E) Seaweed farms would make more money by farming seaweed to sell as nutritional supplements than by farming seaweed to sell as fuel.

-the profit about the plant is irrelevant
3.J:p: the February 1994 opening of Videorama, the first and only video rental outlet in the area that, in addition to renting videos, also          sold them cheaply.
B:p:the decline was on the order of 10,000 rentals.  Yet Videorama sold only 4,000 videos in 1994.
c:Video rental outlets in Centerville together handled 10,000 fewer video rentals in 1994 than in 1993.
maybe some people buy videos instead of renting them because of the cheap price.
(A) In 1994 Videorama rented out more videos than it sold.
--support
(B) In 1994 two new outlets that rent but that do not sell videos opened in Centerville.
--the question has nothing to do with other stores
(C) Most of the video rental outlets in Centerville rent videos at a discount on certain nights of the week.
--the marketing skills are irrelevant
(D) People often buy videos of movies that they have previously seen in a theater.
--it has nothing to do with a theater
(E) People who own videos frequently loan them to their friends.

--loaning the videos to friends may decrease the rentals from stores. and the cheap price increase the number of videos owned by citizens--weaken--right
虽然思路又一次偏离,但还是离答案不远~~~
4.M: b:Danville Winery's new wine was introduced to compete with our most popular wine, which is sold in a distinctive tall, black bottle.       panville uses a similar bottle.  
   c: many customers intending to buy our wine will mistakenly buy theirs instead.
D: p:The two bottles can be readily distinguished
   cpposite to M's
maybe the consumers are careless.
or maybe the D's label color are similar with gold.
(A) Gold is the background color on the label of many of the wines produced by Danville Winery.
--other types of wine are irrelevant
(B) When the bottles are viewed side by side, Danville Winery's bottle is perceptibly taller than Mourdet Winery's.
--weakens M's conclusion, not D's
(C) Danville Winery, unlike Mourdet Winery, displays its wine's label prominently in advertisements.
--irrelevant
(D) It is common for occasional purchasers to buy a bottle of wine on the basis of a general impression of the most obvious feature of the bottle.
--the consumers do not pay attention to the color of the label--right
(E) Many popular wines are sold in bottles of a standard design.

--other types of wine are irrelevant
5.
Finding of a survey of Systems magazine subscribers:  Thirty percent of all merchandise orders placed by subscribers in response to advertisements in the magazine last year were placed by subscribers under age thirty-five.

Finding of a survey of advertisers in Systems magazine:  Most of the merchandise orders placed in response to advertisements in Systems last year were placed by people under age thirty-five.
For both of the findings to be accurate, which of the following must be true?
(A) More subscribers to Systems who have never ordered merchandise in response to advertisements in the magazine are age thirty-five or over than are under age thirty-five.
(B) Among subscribers to Systems, the proportion who are under age thirty-five was considerably lower last year than it is now.
(C) Most merchandise orders placed in response to advertisements in Systems last year were placed by Systems subscribers over age thirty-five.
(D) Last year, the average dollar amount of merchandise orders placed was less for subscribers under age thirty-five than for those age thirty-five or over.
(E) Last year many people who placed orders for merchandise in response to advertisements in Systems were not subscribers to the magazine.
不懂~~~
作者: qiuhua01234567    时间: 2011-11-22 12:53
标题: 帮我看下第五题,我木讷了,版主- -
第三天





1.weaken,30S

background:tide to the size of population,there is little room for growth in the overall market.

Pemise:purchase only once or twice.

Population:gain market share through purchasing competitors.

(A).Most of the major carpet producers market
other floor coverings as well.

-------------------------------------------------irrevelent
(B) Most established carpet producers market
several different brand names and varieties,
and there is no remaining niche in the
market for new brands to fill.

-------------------------------------------------irrelevent
(C) Two of the three mergers in the industry’s last
ten years led to a decline in profits and
revenues for the newly merged companies.

--------------------------------------------------irrelevent
(D) Price reductions, achieved by cost-cutting in
production, by some of the dominant firms
in the carpet market are causing other
producers to leave the market altogether.

----------------------------------------------correct.
(E) The carpet market is unlike most markets in
that consumers are becoming increasingly
resistant to new patterns and styles.

---------------------------------------------irrelevent
2.25S weaken

Backgorund:find ways to reduce CD from atmosphere

Ways:establish farms.when plants die,burn it.

Result:reduce CD

(A)Some areas of ocean in the Southern Hemisphere do not contain sufficient nutrients to support large seaweed farms.

------------------------------------------------------------irrelevent.
(B) When a seaweed plant is burned, it releases an amount of carbon dioxide comparable to the amount it has absorbed in its lifetime.

------------------------------------------------------------correct
(C) Even if seaweed farms prove effective, some people will be reluctant to switch to this new fuel.

---------------------------------------------------------------irrelevent.
(D) Each year about seven billion tons of carbon dioxide are released into the atmosphere but only about five billion tons are absorbed by plant life.

---------------------------------------------------------------even little,already do this.
(E) Seaweed farms would make more money by farming seaweed to sell as nutritional supplements than by farming seaweed to sell as fuel.

-----------------------------------------------------------------have nothing to do with money.

3.weaken30S

J:premisepening of v,rent and sold cheaply

Conclusion:C fewer 1000

B:premise:rent decline 1000,only sold4000

Conclusion:another reason

A)In 1994 Videorama rented out more videos than it sold.

--------------------------------------------------------------------------already say,irrelevent.
(B) In 1994 two new outlets that rent but that do not sell videos opened in Centerville

.
--------------------------------------------------------------------------irrelevent
(C) Most of the video rental outalets in Centerville

rent videos at a discount on certain nights of the week.
-------------------------------------------------------------------------irrelevent
(D) People often buy videos of movies that they have previously seen in a theater.

-------------------------------------------------------------------------irrelevent.
(E) People who own videos frequently loan them to their friends.
-------------------------------------------------------------------------correct

4.weaken 37S

MW:premise:  Danville
Winery's new wine was introduced to compete with our most popular wine, which is sold in a distinctive tall, black bottle.  Danville uses a similar bottle
Conclusion: it is likely that many customers intending to buy our wine will mistakenly buy theirs instead

DW:premise: the label on ours, but not on theirs, is gold colored

Conclusion: The two bottles can be readily distinguished

A)Gold is the background color on the label of many of the wines produced by Danville Winery.

-------------------------------------------------------------asupport
(B) When the bottles are viewed side by side, Danville Winery's bottle is perceptibly taller than Mourdet Winery's.

-----------------------------------------------------------support
(C) Danville Winery, unlike Mourdet Winery, displays its wine's label prominently in advertisements.

-------------------------------------------------------------support
(D) It is common for occasional purchasers to buy a bottle of wine on the basis of a general impression of the most obvious feature of the bottle.

-------------------------------------------------------------correct
(E) Many popular wines are sold in bottles of a standard design.

------------------------------------------------------------irrelevent.

5. Must be true 38S

(A) More subscribers to Systems who have never ordered merchandise in response to advertisements in the magazine are age thirty-five or over than are under age thirty-five.
(B) Among subscribers to Systems, the proportion who are under age thirty-five was considerably lower last year than it is now.
(C) Most merchandise orders placed in response to advertisements in Systems last year were placed by Systems subscribers over age thirty-five.
(D) Last year, the average dollar amount of merchandise orders placed was less for subscribers under age thirty-five than for those age thirty-five or over.
(E) Last year many people who placed orders for merchandise in response to advertisements in Systems were not subscribers to the magazine.

求分析。。。。

作者: winghyy    时间: 2011-11-22 13:46
1. 48s-weaken
BG:there is liittle room for the growth of carpet market because of the size of population.
Premise: most people only purchse carpets once or twice in their life.
Conclusion: If the carpet company can only gain the market share by purchasing competitor company, not by the aggressive marketing.
Prephase: The carpet company can launch new creative designs to attract consumers.
D
(A) Most of the major carpet producers market
other floor coverings as well. ——irrelevant(floor covering)
(B) Most established carpet producers market
several different brand names and varieties,
and there is no remaining niche in the
market for new brands to fill. ( ireelevant-new brand)
(C) Two of the three mergers in the industry’s last
ten years led to a decline in profits and
revenues for the newly merged companies. ——irrelevant(profits and revenues)
(D) Price reductions, achieved by cost-cutting in
production, by some of the dominant firms
in the carpet market are causing other
producers to leave the market altogether.  ——correct (provides a way of marketing)
(E) The carpet market is unlike most markets in
that consumers are becoming increasingly
resistant to new patterns and styles. ——strengthen

2. 50s-weaken
Premise: Scientists are disscussing about how to reduce the CO2 by the increasing absorption of plant.
Conclusion: They plan to establish seaweed farms in the oceans. When the seaweed dies, it will be disposed as fuel.
Prephase:When the seaweed dies, it will be burned and produce more CO2
B

3. 65s-weaken (complex argument, weaken the objection of B)
Premise:    J: the overall video rentals decreased 10000 in 1994 than in 1993, because a new outlet opened in 1994 and it sold videos cheaply.
Conclusion: B: the decline can not be explained by the opening of the new outlet. Because it only sold 4000 videos in 1994.
Prephase: ?? People who rent videos always rent videos more than once per rental?
E

4. 46s-weaken (weaken D)
Premise: M: D introduced a new wine to compete our most popular wine, which is sold in distinctive tall and black bottle. So M's consumers will mistakenly buy D's wine instead.
D:   Objection——the lable on the bottle is different. D's is gold.
Prephase: Consumers pay more attention on the shape of the bottle and the gold under the light in cann't be seen.
D

5. 60s-must be true
Premise: Report 1 :30% subscribers placed order in response to the ad are under 35 years old.
               Report 2: most of the orders placed in response to the ad are under 35 years old
E

A) More subscribers to Systems who have never ordered merchandise in response to advertisements in the magazine are age thirty-five or over than are under age thirty-five.【wrong, only 30% subscribers order and we don't know the percentage of that age under 35】
(B) Among subscribers to Systems, the proportion who are under age thirty-five was considerably lower last year than it is now. 【No statistics for this year】
(C) Most merchandise orders placed in response to advertisements in Systems last year were placed by Systems subscribers over age thirty-five. 【only 30%】
(D) Last year, the average dollar amount of merchandise orders placed was less for subscribers under age thirty-five than for those age thirty-five or over. 【not mention about the average amount】
(E) Last year many people who placed orders for merchandise in response to advertisements in Systems were not subscribers to magazine. 【Correct: 30% vs most】


作者: Jane412    时间: 2011-11-22 14:28
精炼[weaken]
Time:33’’
Premise: the carpet market is tied to the size of the population
Premise: most consumers buy carpet twice, first in twenties or thirties, and then in fifties or sixties.
Premise: carpet companies purchase competitors
Conclusion: then companies can gain market share in the carpet market
A) Most of the major carpet producers market other floor coverings as well.
------irrelevant
(B) Most established carpet producers market
several different brand names and varieties,
and there is no remaining niche in the
market for new brands to fill.
----irrelevant------------>市场饱和,那么只有并购来增加市场份额,support
(C) Two of the three mergers in the industry’s last
ten years led to a decline in profits and
revenues for the newly merged companies.
------irrelevant------------》shell game+conditional statement
(D) Price reductions, achieved by cost-cutting in
production, by some of the dominant firms
in the carpet market are causing other
producers to leave the market altogether.
-----correct
(E) The carpet market is unlike most markets in
that consumers are becoming increasingly
resistant to new patterns and styles.
-------irrelevant

++++++++++++++++++++++++++++++++++++
看完解释发现自己对题目理解的太浅薄了。。。

2[weaken]
Time:38’’
Premise: increaseing the amount that is absorbed by plant can remove excess co2
Premise: estabilish seaweed farms in the oceans and burn then for fuel
Conclusion: this method can remove excess co2
Prephrase:燃烧之后产生的co2≥被吸收掉的
(A) Some areas of ocean in the Southern Hemisphere do not contain sufficient nutrients to support large seaweed farms.
--------irrelevant
(B) When a seaweed plant is burned, it releases an amount of carbon dioxide comparable to the amount it has absorbed in its lifetime.
--------------correct!
(C) Even if seaweed farms prove effective, some people will be reluctant to switch to this new fuel.
-----------irrelevant。愿不愿意和有木有效是两回事儿
(D) Each year about seven billion tons of carbon dioxide are released into the atmosphere but only about five billion tons are absorbed by plant life.
-------------irrelevant。不一定是这个方法的效果。即便是这个方法的,也不能通过绝对数的比较判定有木有效果。可能别的方法去掉的co2更少。
(E) Seaweed farms would make more money by farming seaweed to sell as nutritional supplements than by farming seaweed to sell as fuel.
-----------------irrelevant。Make money和题目无关

3[weaken]
Time: 43’’
Premise:Videorama opened in 1994
Premise: videos were sold and rented cheaply in V
Premise: video rental outlets together handled 10000 fewer rentals in 1994 than 1993
Conclusion: V should responsible for this.
BUT
Premise:Videorama sold 40000videos in 1994
Premise: the decline was 10000
Conclusion: Videorama didn’t responsible for the declination
(不清楚这种反驳题型的premise-conclusion怎么写。。。)

Prephrase:大部分rent的生意也被V抢了

A) In 1994 Videorama rented out more videos than it sold.
----------correct!--------------》只说Videorama租的比卖的多不能说明租的加卖的超过10000,如果不超过还是说明有其他原因。模棱两可
(B) In 1994 two new outlets that rent but that do not sell videos opened in Centerville.
-------------irrelevant
(C) Most of the video rental outlets in Centerville rent videos at a discount on certain nights of the week.
---------irrelevant
(D) People often buy videos of movies that they have previously seen in a theater.
------------irrelevant
(E) People who own videos frequently loan them to their friends.
------------support-------------》因为Videorama,人们买,而买的人又转租给别人,使得别人不再从Video rental outlets租借,因此租的少了,起始原因还是Videorama

4[weaken]
Time:28’’
Premise’s bottle is similar to ours
Conclusion: consumers will mistakely buy D’s when they want to buy ours

BUT
Premise: the labels are different
Conclusion: consumers can distinguish them

Prephrase: 如果标签不明显或颜色容易混淆,顾客还是会拿错

(A) Gold is the background color on the label of many of the wines produced by Danville Winery.
----------------------------irrelevant
(B) When the bottles are viewed side by side, Danville Winery's bottle is perceptibly taller than Mourdet Winery's.
---------------------------------support
(C) Danville Winery, unlike Mourdet Winery, displays its wine's label prominently in advertisements.
----------------------------------support
(D) It is common for occasional purchasers to buy a bottle of wine on the basis of a general impression of the most obvious feature of the bottle.
----------------------------------correct!
(E) Many popular wines are sold in bottles of a standard design.
--------------------------------irrelevant

5[support]
Time:38’’
Premise:30% merchandise orders were placed by subscribers under age 35
Premise: most orders were placed by people under age 35
(这题画个图做的~)
(A) More subscribers to Systems who have never ordered merchandise in response to advertisements in the magazine are age thirty-five or over than are under age thirty-five.
----------wrong
(B) Among subscribers to Systems, the proportion who are under age thirty-five was considerably lower last year than it is now.
-------------irrelevant
(C) Most merchandise orders placed in response to advertisements in Systems last year were placed by Systems subscribers over age thirty-five.
----------wrong
(D) Last year, the average dollar amount of merchandise orders placed was less for subscribers under age thirty-five than for those age thirty-five or over.
-----------wrong
(E) Last year many people who placed orders for merchandise in response to advertisements in Systems were not subscribers to the magazine.
--------correct
作者: Rachel1991    时间: 2011-11-22 15:10
要回复才能看见选项吗?
作者: Jane412    时间: 2011-11-22 15:37
要回复才能看见选项吗?
-- by 会员 Rachel1991 (2011/11/22 15:10:57)



选项设成白色了,只要在每个题目下面空白处drag一下就能看到选项了~~
作者: Jane412    时间: 2011-11-22 15:46
呃。。。我明天的作业好好调格式。。。。空行太占版面了。。。
作者: trackofheart    时间: 2011-11-22 16:02
background: There is relatively little room for growth in the overall carpet market, which is tied to the size of the population.
premise:Most who purchase carpet do so only once or twice, first in their twenties or thirties, and then perhaps again in their fifties or sixties.
conclusion:
Thus as the population ages, companies producing carpet will be able to gain market share in the carpet market only through purchasing competitors, and not through more aggressive marketing.

整体:
20、30 岁买一次,50、60岁买一次 ——》gain market share in the carpet market only through purchasing competitors, and not through more aggressive marketing.

pre:每年不都是有新一批的人长大2、30岁,5、60岁么?(请问我的pre跟答案一点关系都没有,大家都是怎么pre的啊?谢谢)

选了:d  (是那种看了答案觉得应该是d)

选项分析:
a:market other floor covering as well 与 through 什么来gain market share无关
b:market several brands 也不会对gain market share产生影响,因为它是受人口限制的,人们只是在2、30和5、60岁的时候买一次。。。
c:mergers 三个中的两个导致newly merger利润和收入下降与gain market share无关。
d:主要的firm通过cut price让一些厂家离开了。。这是gain market 的另一手段。weaken了结论
e:消费者对新pattern的抵制,但是还是在2、30和5、60岁的时候买一次。。。对结论gain market share没有什么影响。。。



2、
目的:to remove excess carbon dioxide from the atmosphere
方法:to establish giant floating seaweed farms in the oceans.  When the seaweed plants die, they will be disposed of by being burned for fuel.


pre:seaweed被burn了以后产生的carbon dioxide肿么办呢


a:在一定程度上不能贯彻plan的实施,削弱,没有b强
b:最削弱
c:some people在plan中不起作用,起作用了也是削弱plan的实施,但没有b强
d:范围太大,且跟plan无关
e:对题目所给条件的驳斥,题目已经说了they will be disposed of by being burned for fuel.而且即使这样的话,也是利于plan的实施,是增强。。


3、1’13‘’
没有感觉?


4、
p:Danville uses a similar bottle.  Thus, it is likely that many customers intending to buy our wine will mistakenly buy theirs instead.
c:The two bottles can be readily distinguished:  the label on ours, but not on theirs, is gold colored.


pre:关于区分bottle的方法影响消费者的决定


a:更加显示出两者区别,增加
b:同上
c:同上
d:影响消费者决定的因素与dw的说法不一致,削弱
e:无关


5、
没有感觉


请问下大家:想第3、5题这种题目读完了没有任何感觉。也不知道思路是什么。。要肿么练习呢。。。谢谢
作者: balapupu    时间: 2011-11-22 17:12
来做作业喽~~看这这么高的人气还是很开心的说哈~~吼吼~~~
1.【40s】【前提结论型】
P:the room of the market for the C Company to expand is hard-->people only buy twice C product in their whole life.-->the market share is tied to the population.
C:there is no way to improve the market share only through defeat other competitors.
weaken: there is a report that in the future the population age from 20-60 will more than the past.[find reason to support that the marking in fact is aggressive]
A:

A) Most of the major carpet producers market
other floor coverings as well.-->irrelevant with the floor
(B) Most established carpet producers market
several different brand names and varieties,
and there is no remaining niche in the
market for new brands to fill.--> irrelevant wit the new brands.【X-->support】
(C) Two of the three mergers in the industry’s last
ten years led to a decline in profits and
revenues for the newly merged companies.-->support【market share<-->profits】
(D) Price reductions, achieved by cost-cutting in
production, by some of the dominant firms
in the carpet market are causing other
producers to leave the market altogether.-->support【R】
(E) The carpet market is unlike most markets in
that consumers are becoming increasingly
resistant to new patterns and styles.-->R【x--> support,连选项的意思都搞错了,泪奔啊。。。】
LSAT的逻辑真是做的有够重伤的,想起以前抓那个lsat的阅读的时候也是。。。。。
哎哎。。。总结下我这道题的错误点:
首先还是结论重心木有抓住,攻击点攻击错了,是要攻击通过并购这种方式并不是唯一的增加市场份额的选择,而不是攻击市场不够aggressive。。。哎。。想偏到十万八千里了。
C选项真的很阴险,还是要紧扣结论关系才能作对,就是方式:并购——》目的:扩大市场份额的关系。
作者: qiuhua01234567    时间: 2011-11-22 21:35
求进群-  -,我是好孩子
作者: balapupu    时间: 2011-11-22 22:15
继续:逻辑链:
2.【25s】【措施目的型】
P:in order to remove the CO2 in the atmosphere-->increase the amount absorbed by the plant-->C:plant  seaweed-->when the seaweed die,people can burn it for fuel.
weaken the plan: burning the fuel will induce more Co2.
A:
(A) Some areas of ocean in the Southern Hemisphere do not contain sufficient nutrients to support large seaweed farms.—>just some areas can not prove the plan is not good.
(B) When a seaweed plant is burned, it releases an amount of carbon dioxide comparable to the amount it has absorbed in its lifetime.-->R
(C) Even if seaweed farms prove effective, some people will be reluctant to switch to this new fuel.-->same as A.
(D) Each year about seven billion tons of carbon dioxide are released into the atmosphere but only about five billion tons are absorbed by plant life.-->though the amount is small, the plan is still efficient.
(E) Seaweed farms would make more money by farming seaweed to sell as nutritional supplements than by farming seaweed to sell as fuel.-->irrelevant with profits of farms.

3.[27s]
j:V rental outlets loss 10000 video rentals sell in 1994-->the reason is because the new Rama video outlets,it both sell and rental videos.
B:J is wrong, because Rama only sold 4000 videos in 1994
weaken B: many people buy the video will lead the video to others.
A:
(A) In 1994 Videorama rented out more videos than it sold.-->irrelevant comparison.
(B) In 1994 two new outlets that rent but that do not sell videos opened in Centerville.-->irrelevant with other outlets.
(C) Most of the video rental outlets in Centerville rent videos at a discount on certain nights of the week.-->irrelevant with the time
(D) People often buy videos of movies that they have previously seen in a theater.-->support.
(E) People who own videos frequently loan them to their friends.-->R

4.[27s]
MW: the bottle of the DW wine is as the same as the MW-->it is likely that the consumer who want to buy the MW wine will mistakenly but the DW wine.
DW: the label of the DW is different-->so the consumer can figure out.
weaken DW: consumer often judge the wine only through the shape.
A:
(A) Gold is the background color on the label of many of the wines produced by Danville Winery.-->support
(B) When the bottles are viewed side by side, Danville Winery's bottle is perceptibly taller than Mourdet Winery's.-->support
(C) Danville Winery, unlike Mourdet Winery, displays its wine's label prominently in advertisements.-->support
(D) It is common for occasional purchasers to buy a bottle of wine on the basis of a general impression of the most obvious feature of the bottle.-->R
(E) Many popular wines are sold in bottles of a standard design.-->irrelevant with the popularity.

5.[1'00][一开始看以为是一句话说了两遍]
subscribers:30% M orders in placed by subscribers under 35.
advertisers: most of M orders is placed by people under 35.
must be true.the subscribers under 35 is fewer than other age group.[这道题让我想到汽车超速罚款那道题]
A:
(A) More subscribers to Systems who have never ordered merchandise in response to advertisements in the magazine are age thirty-five or over than are under age thirty-five.-->can not infer the never order group.
(B) Among subscribers to Systems, the proportion who are under age thirty-five was considerably lower last year than it is now.-->irrelevant comparison.
(C) Most merchandise orders placed in response to advertisements in Systems last year were placed by Systems subscribers over age thirty-five.--> can not conclude the orders change.
(D) Last year, the average dollar amount of merchandise orders placed was less for subscribers under age thirty-five than for those age thirty-five or over.--> irrelevant with the money
(E) Last year many people who placed orders for merchandise in response to advertisements in Systems were not subscribers to the magazine.-->R
作者: chensong211    时间: 2011-11-22 22:54
精炼题:
Premise: most people purchase carpet do once or twice. So the market of carpet is limited
Conclusion: improve the company’s market of carpet only though purchase competitors.
Logic: the size population is limited = the purchase only once or twice  =  the market is limited  = must through purchase competitors to improve the company’s market.
Infer: increase the size population or change 购买次数
A. 无关,偏题了没有说到carpet市场
B. 无关,没有讲到改变市场。
C. 无关
D. bingo 通过挤走竞争对手一样可以占领市场。
E. support
作者: fox0923    时间: 2011-11-23 00:36
求进群-  -,我是好孩子
-- by 会员 qiuhua01234567 (2011/11/22 21:35:40)


一个系列下来坚持住就可以进群了~~
嘻嘻~不过进了群也是要坚持练习的~加油,好孩子
作者: qiuhua01234567    时间: 2011-11-23 09:25
求进群-  -,我是好孩子
-- by 会员 qiuhua01234567 (2011/11/22 21:35:40)



一个系列下来坚持住就可以进群了~~
嘻嘻~不过进了群也是要坚持练习的~加油,好孩子
-- by 会员 fox0923 (2011/11/23 0:36:35)


恩恩,明了。你貌似很积极啊,天天看见的沙发
作者: zz42050524    时间: 2011-11-23 12:25
2011/11/23
精炼:Weaken 50s
P: The population size limits the market of carpet, because people always buy carpet when they are 20s to 30s or 50s to 60s.
C: The producer of carpet have to compete to gain market shares rather than through aggressive marketing.
推测:Some other possible ways can broaden the market size of carpet.
A) Most of the major carpet producers market  The stimulus only concern the carpet market.
other floor coverings as well.
(B) Most established carpet producers market   This is support. Compete with potential rivals.
several different brand names and varieties,
and there is no remaining niche in the
market for new brands to fill.
(C) Two of the three mergers in the industry’s last  irrelative
ten years led to a decline in profits and
revenues for the newly merged companies.
(D) Price reductions, achieved by cost-cutting in  That’s it.
production, by some of the dominant firms
in the carpet market are causing other
producers to leave the market altogether.
(E) The carpet market is unlike most markets in  irrelative
that consumers are becoming increasingly
resistant to new patterns and styles.
D
逻辑链
2.Weaken 30s
P: Scientist will establish giant seaweed farms in ocean and burn the seaweeds when they are ripe.
C; The carbon dioxide will reduce in the atmosphere.

推测:burning will produce more or less carbon dioxide that the seaweeds have absorbed.

B

3.weaken 50s
J: The rentals of video in 1994 are 1000 less than that in 1993, because a xxx opened in February 1994, in addition to renting videos ,it sell the videos cheaply.
B: There must be other explanation. In 1994, it sell 400 videos while the rentals reduce 1000.
推测:people who buy videos would lent them to others.
E

4.Weaken 40s
M; The bottle of J’s wine is similar to ours, it has the same color and is only a little small. Customers who want to buy our wine easily mistaken to buy J’s.
J: The label of our wine is different, it readily to distingulish.

推测: The label can’t be easily distingulish.

D 推测错误

5 MUST BE TRUE 50S
这道题记不住 推理也需要询问
作者: 风无衣    时间: 2011-11-23 20:48
天呐!!!我好不容易写到最后一道题,手贱啊!!!木有了!!!又重新写~~~哎……

【精炼1-3】
weaken_1min15s
background informationverall carpet market has little room for growth
premise:most purchase crapet only once or twice. first in 20 or 30. then in 50 or 60
conclusion:companies producing carpet will be able to gain market share in the carpet market only through purchasing competitors, and not through more aggressive marketing.

(A) Most of the major carpet producers market
other floor coverings as well.——irrelevant
(B) Most established carpet producers market
several different brand names and varieties,
and there is no remaining niche in the
market for new brands to fill.——support
(C) Two of the three mergers in the industry’s last
ten years led to a decline in profits and
revenues for the newly merged companies.——并购新企业利润和收入下降~说明并购企业often financially unsuccessful~但是木有weaken结论~
(D) Price reductions, achieved by cost-cutting in
production, by some of the dominant firms
in the carpet market are causing other
producers to leave the market altogether.——correct
(E) The carpet market is unlike most markets in
that consumers are becoming increasingly
resistant to new patterns and styles.——irrelevant

【逻辑链1-3】
2、weaken_27s
目标:通过增加植物吸收carbon dioxide,从而减少carbon dioxide的量
计划:在海上建立seaweed farm。当seaweed死后燃烧获取fuel~
weaken:燃烧产生的carbon dioxide 量与其吸收carbon dioxide的量相当,甚至更多~
B

3、47s
J:C地1994年比1993年video出租量减少是由于videorama的开张。不仅出租video,且低价贩卖
  出租量一共减少10000
B:肯定还有其他原因。因为videorama只卖了4000,但减少了10000
weaken B。找到选项说明减少10000就是由于videorama的开张造成的
E

4、27s
MW:DW的新酒瓶子和我们家最受欢迎的酒瓶子长得差不多,都是高的黑瓶子~所以有可能顾客本来想买我们家的酒却错拿了他们家的瓶子
DW:不太可能。因为两家的酒瓶子还是有区别的:我们家的label是金色的,他家不是~
weaken DW。找到选项说明顾客确实有可能拿错。
D

5、must be true_1min8s看了好几遍~
S杂志订阅者调查:在去年回应杂志广告订阅者中,30%的所有商品订单placed by35岁以下的订阅者
S杂志广告商调查:在去年回应杂志广告的,most商品订单placed by35岁以下的人
E
作者: 小意达de花儿    时间: 2011-11-24 17:00
1. Carpet market, which related to the size of population, is little
Many people buy carpet once or twice: 20-30, 50-60
CONCLUSION: In order to gain market share, carpet company should purchase competitors, instead aggressive market.
I GUESS: Population is growing; Purchasing competitor has disadvantage; Aggressive marketing has advantage
(A) Most of the major carpet producers market other floor coverings as well. IRRELEVANT
(B) Most established carpet producers market several different brand names and varieties, and there is no remaining niche in the market for new brands to fill. IRRELEVANT
(C) Two of the three mergers in the industry’s last ten years led to a decline in profits and revenues for the newly merged companies. Advantage for newly merged company
(D) Price reductions, achieved by cost-cutting in production, by some of the dominant firms in the carpet market are causing other producers to leave the market altogether. This is the advantage of aggressive marketing
(E) The carpet market is unlike most markets in that consumers are becoming increasingly resistant to new patterns and styles. Just a supplemental imformation

2. Burning seaweed plants produces more carbon dioxide
(A) Some areas of ocean in the Southern Hemisphere do not contain sufficient nutrients to support large seaweed farms. Insufficient because of ‘some areas’
(B) When a seaweed plant is burned, it releases an amount of carbon dioxide comparable to the amount it has absorbed in its lifetime. CORRECT
(C) Even if seaweed farms prove effective, some people will be reluctant to switch to this new fuel. Insufficient because of ‘some people’
(D) Each year about seven billion tons of carbon dioxide are released into the atmosphere but only about five billion tons are absorbed by plant life. IRRELEVANT
(E) Seaweed farms would make more money by farming seaweed to sell as nutritional supplements than by farming seaweed to sell as fuel. IRRELEVANT---money

3. Some people who buy videos perhaps lent those video to their friends.
(A) In 1994 Videorama rented out more videos than it sold. This comparison doesn’t help
(B) In 1994 two new outlets that rent but that do not sell videos opened in Centerville. IRRELEVNAT
(C) Most of the video rental outlets in Centerville rent videos at a discount on certain nights of the week. IRRELEVANT
(D) People often buy videos of movies that they have previously seen in a theater. IRRELEVANT
(E) People who own videos frequently loan them to their friends. CORRECT
4. CUSTOMS cannot distinguish this difference
(A) Gold is the background color on the label of many of the wines produced by Danville Winery. IRRELEVANT
(B) When the bottles are viewed side by side, Danville Winery's bottle is perceptibly taller than Mourdet Winery's. SUPPORT
(C) Danville Winery, unlike Mourdet Winery, displays its wine's label prominently in advertisements. SUPPORT
(D) It is common for occasional purchasers to buy a bottle of wine on the basis of a general impression of the most obvious feature of the bottle. CORRECT
(E) Many popular wines are sold in bottles of a standard design. IRRELEVANT
5. People who place order in response to ads in Sys did not subscriber the magazine
作者: daisyの小夢想    时间: 2011-11-24 23:10
求进群-  -,我是好孩子
-- by 会员 qiuhua01234567 (2011/11/22 21:35:40)




一个系列下来坚持住就可以进群了~~
嘻嘻~不过进了群也是要坚持练习的~加油,好孩子
-- by 会员 fox0923 (2011/11/23 0:36:35)



恩恩,明了。你貌似很积极啊,天天看见的沙发
-- by 会员 qiuhua01234567 (2011/11/23 9:25:15)



 因为时差,truf你就不要想着和foxxy抢沙发了哈哈
作者: ugly5552000    时间: 2011-11-24 23:21
1/36’’  D
Background: because the size of population, the room left for the growth of carpet market is so little. Most people only buy carpet only once or twice through their life.
Premise: in order to gain market, the carpet companies only need to buy the competitors, not through the aggressive marketing.
Prephrase: what if the consumers are tired of the tedious styles, and turn to the foreign carpet manufacturer?
A: whether most of the major carpet producers market other floor coverings is irrelevant to the argument.
B: if there is no room for the new brands, it makes sense that purchasing competitors to gain market share, and to market aggressively is not that necessary. This supports the argument, not weakens it.
C: if only the newly merged companies suffered loss of revenue, while the companies merged for year kept benefit, the strategy of purchasing competitors works, then ,the argument is supported.
D: as many companies faded out from the carpet market, the market will be rearranged, the market will be divided by the dominant firms, there is no need to buy the competitors. The argument is weakened.
E: if the consumers are only interested in the old patterns and styles, then the only way to gain market share is to purchase competitors. The argument is supported.
2/39’’   B
Background: scientists are trying to remove excess carbon dioxide from the atmosphere by increasing the amount that is absorbed by plant life.
Premise: one proposition is to found immense floating seaweed farms in the oceans, and burn them for fuel when they die.
Prephrase: during their growth, the carbon dioxide is also let out, when they are burned, the large amount of carbon dioxide will comes out.
A: the atmosphere is a whole body, and incapability will be compensated by the farms in other ocean areas, so it is not the weakness of the plan.
B: this outcome signifies that the plan does nothing change to the total amount of carbon dioxide in the atmosphere.
C: the point is the carbon dioxide, not whether the fuel will be accepted.
D: this is irrelevant to the excess carbon dioxide in the atmosphere.
E: this does not support or weaken the plan.
3/59’’  E
Background: the number of rented videos in 1994 is 10000 fewer than that in 1993.
Premise: J said the decline was due to the opening of Videorama, which only provided video rental, but also sold video. B said as Videorama only sold 4000 videos, there must be other explanation to the decline.
Prephrase: the sold videos are lent to other people by the consumer, this make the number of people renting videos declines.
A: it is paradox to J’s argument.
B: this is the other explaination.
C: it will promote the video rental, not consistent with the fact.
D: it is personal habit, irrelevant to the argument.
E: many people are able to borrow videos from their friends, then less people will rent video from the shop.
4/50’’   D
Background: D’s new wine was introduced to compete with the most popular wine of another company. Their bottles are alike.
Premise: M thinks that customers will mistakenly buy the competitor’s wine. D thinks that the gold colored label on the bottles will make their wine distinguished by the customers.
Prephrase: most customers who frequently buy the wines never pay attention to the labels. Both the bottle and the label are similar.
A: no comparison, irrelevant.
B: the argument is about the label, not the shape of bottles
C: this supports the argument, not weakens it.
D: If it is true, whether the label is distinguished or not, the consumer will not much realizes that, the label-method is not useful
E: irrelevant.
5/E
Background: last year, 30% of the merchandise orders in response to Ads were placed by subscribers under age 35, most of the merchandise orders in response to Ads were placed by people under age 35.
Prephrase: many non-subscribers under age 35 have placed the merchandise orders to Ads.
A: irrelevant
B: both the survey figures were in last year, irrelevant.
C: conflict to the survey.
D: irrelevant
E: this is the only explanation
作者: UlysessHope    时间: 2011-12-24 23:18
【精练】
p: carpet market is tied to the size of population. some people buy carpets only once or twice
c: carpet compenies can be gain market share only by acquisition instead of more aggressive marketing
weaken:
(A) other floor coverings as well--irrelevant
(B) Most established carpet producers market
several different brand names and varieties,
and there is no remaining niche in the
market for new brands to fill.---strengthen
(C) Two of the three mergers in the industry’s last
ten years led to a decline in profits and
revenues for the newly merged companies.-----irrelevant 与利润无关,看share
(D) Price reductions, achieved by cost-cutting in
production, by some of the dominant firms
in the carpet market are causing other
producers to leave the market altogether.-----weaken 说明第二种方法可行
(E) new patterns and styles.---irrelevant
【逻辑链】
2.
(A) irrelevant
(B) When a seaweed plant is burned, it releases an amount of carbon dioxide comparable to the amount it has absorbed in its lifetime.---weaken
(C) irrelevant
(D) irrelevant
(E) irrelevant
3.
(A) In 1994 Videorama rented out more videos than it sold. ----irrelevant 没说总的多少
(B) irrelevant
(C) strengthen
(D) irrelevant
(E) People who own videos frequently loan them to their friends.----weaken
4.
(A) irrelevant
(B) strengthen/irrelevant
(C) irrelevant
(D) It is common for occasional purchasers to buy a bottle of wine on the basis of a general impression of the most obvious feature of the bottle.----weaken
(E) irrelevant
5.
Finding of a survey of Systems magazine subscribers:  Thirty percent of all merchandise orders placed by subscribers in response to advertisements in the magazine last year were placed by subscribers under age thirty-five.
所有由订户定的订单的30%的订户在35岁以下
Finding of a survey of advertisers in Systems magazine:  Most of the merchandise orders placed in response to advertisements in Systems last year were placed by people under age thirty-five.
由所有人订阅的订单中大部分(超过50%)人在35岁以下
For both of the findings to be accurate, which of the following must be true?
-- PREP07 CR1 NO.12
(A) More subscribers to Systems who have never ordered merchandise in response to advertisements in the magazine are age thirty-five or over than are under age thirty-five. 跟never ordered无关
(B) Among subscribers to Systems, the proportion who are under age thirty-five was considerably lower last year than it is now.   跟last year无关
(C) Most merchandise orders placed in response to advertisements in Systems last year were placed by Systems subscribers over age thirty-five.   大部分订单由年龄超过35的订户订下,与第二个调查冲突
(D) Last year, the average dollar amount of merchandise orders placed was less for subscribers under age thirty-five than for those age thirty-five or over. 跟dollar amount 无关
(E) Last year many people who placed orders for merchandise in response to advertisements in Systems were not subscribers to the magazine. 大部分订单不是订户订的,解释了两个调查
作者: hexiaomu77    时间: 2011-12-25 13:11
5.第一个:30%的 merchandise orders placed by subscribers在35一下
第二个:大部分的merchandise orders in response to advertisements in Systems都在35一下
结论:包含关系
前一个是30%后一个是most,可见后一个是包含前一个的,后一个里有前一个没有的 即是答案E
(E) Last year many people who placed orders for merchandise in response to advertisements in Systems were not subscribers to the magazine.
作者: lan0604    时间: 2012-2-13 21:30
精炼:
00:45
weaken

carpet consumption with ages--->carpet manufactures compete against competitors not through marketing
marketing can work
C错了,D正确

2
00:51
plant seaweed in oceans then burn for fuel--->remove excess carbon dioxide
burning may release more carbon dioxide
B

3
01:07
weaken
Videorama sold 4,000 while others decrease d 10,000--->there are other explanations
....
A,貌似错了

4
00:32
weaken
labels are good colored---> won't be mistaken
...
D

5
01:09
没懂吖

求斑竹写一下逻辑链的答案吖~还是我木有找到?
作者: leewonting    时间: 2012-4-2 20:12
27
背景:Little room for growth in the overall carpet market
条件:Most people purchase carpet d only once or twice, in their twenties and/or fifties
结论:Thus as the population ages, companies producing carpet will be able to gain market share in the carpet market through purchasing competitors, not through more aggressive marketing
推测:there is something disadvantaged in purchasing competitor’s products
选D,除了收购外,降低成本让carpet producer成功

28
Finding of a survey of Systems magazine subscribers:  Thirty percent of all merchandise orders placed by subscribers in response to advertisements in the magazine last year were placed by subscribers under age thirty-five.(在看了广告之后下订单的人中,30%是35岁以下的)
Finding of a survey of advertisers in Systems magazine:  Most of the merchandise orders placed in response to advertisements in Systems last year were placed by people under age thirty-five.(大部分看了广告而下订单的人中,是35岁以下的)
For both of the findings to be accurate, which of the following must be true?
(A) More subscribers to Systems who have never ordered merchandise(超出讨论范围) in response to advertisements in the magazine are age thirty-five or over than are under age thirty-five.
(B) Among subscribers to Systems, the proportion who are under age thirty-five was considerably lower last year than it is now.(超出讨论范围)
(C) Most merchandise orders placed in response to advertisements in Systems last year were placed by Systems subscribers over age thirty-five.(Wrong,应该是35岁以下的人)
(D) Last year, the average dollar amount of merchandise orders placed was less for subscribers under age thirty-five than for those age thirty-five or over.(average dollar超出)
(E) Last year many people who placed orders for merchandise in response to advertisements in Systems were not subscribers to the magazine.
背景:30% of all merchandise orders placed by subscribers in response to ads in the magazine last year were placed by subscribers under age 35
条件:Most of the orders placed in response to ads in Systems last year were placed by people under age 35
结论:
推测:
选C,没读懂题目
应该选E,首先题目中百分比和MOST有出入。(句子意思没有读懂啊!!!)


其他的都做过了~~
作者: Rena张    时间: 2012-4-5 17:16
1.
Time: 1'16
Premise: the size of the carpet companies' market depends on the population. Many people buy carpet only once or twice, in their 20-30 or 50-60.
Conclusion: as the population ages, the carpet company can gain market share only through purchasing competitors, not through more aggressive marketing.
Type: casts doubt on the conclusion.
Prephrase: aggressive marketing has improved profits of some carpet companies.
Choice: D

2.
Time: 45''
Premise: scientists want to increase the amount of carbon absorbed by plants.  
       They plan to plant seaweeds on the surface of the ocean.
       After the seaweeds die, they will be disposed by buring.
Type: weakness of the plan.
Prephrase: the buring of the seaweeds will exhaust more carbon dioxin.
Choice: B

3.
Time: 1'48''
J: Premise: video rental declined 10,000 in 1994 than in 1993.
  Conclusion:The decline may due to the opening of V, who sold video rental outlet cheaply.
B: V sole only 4000 videos in 1994.
Type: weaken B's xxx
Prephrase: people who buy the videos may lent them to their friends.
Choice: E

4.
Time: 41''
M: Premise: D use a similar tall and black bottle to M's
  Conclusion: customers who intend to buy M's wine may mistakenly buy D's.
D: D's label on the bottle is gold colored which can be distinguished.
Type: undermines D's xxx
Prephrase: the label on the bottle is too small to be noticed.
Choice: D

5.
Time: 2'
Premise: 1, 30% of XXX is placed by subscribers under 35.
       2, most of XXX is placed by people under 35.
Type: must be true
Prephrase: most of these people under 35 are subscribers.
Choice: E
作者: Rena张    时间: 2012-4-5 17:17
做作业的速度太慢太慢啦~~~慢慢慢慢慢慢a
作者: lixiang8747    时间: 2012-4-5 21:13
分析一道题的时间好久
作者: lixiang8747    时间: 2012-4-5 21:18
1
BG: There is relatively little room for growth in the overall carpet market, which is tied to the size of the population.
premise:Most who purchase carpet do so only once or twice.
conlusion:Thus as the population ages, companies producing carpet will be able to gain market share in the carpet market only through purchasing competitors, and not through more aggressive marketing.
Prephrase:公司有其他方式来盈利
选D
A 无关
B  好像加强了吧
c 新并购的利润降低和收入降低不能说明只有并购才是出路
d 讲出了集中提高收入的方式
e  无关
作者: xunjiejie5    时间: 2012-4-15 15:41
1 前提:market of carpet has a relationship with the population. And people always buy carpets twice in their lives.
Conclusion: the opening of the market is to buy other companies
Predicted answers: This way did not work (其他原因导致这个结果不可能发生)
A other floor coverings (wrong)
B support (opposite answer)
C out of the scope
D correct
E out of the scope
2 措施目的型 (措施达不到目的)
Premise: seaweed absorb the C, after seaweed dies, using it for buring.
Conclusion: remove C from the sky
Predicted: this way can not remove C from the sky
A SH the place ( specially not the general in stimulus)
B correct
C support
D support
E 无关比较
3 objection B to J
J: the decline of 10000 is the reason that the opening of the outlet rents and sells videos
B: the reasons are wrong. The sales of the outlet are only 4,000.
Predict: E
5 S: 35%的购物在杂志上的广告是35岁以下的人
 A: 大多数在杂志上购物的是35岁以下的人
 E (说明调查的样本对象是不同的情况,所以并不相矛盾)
作者: zly2011    时间: 2012-4-16 11:21
1、55s weaken
premise people will buy carpet only once or twice. it is due to their ages
conclusion  only through purchasing competitors to gain market share
推测 other ways to attract customes
选项分析 A、无关 most 模糊词 B、new brand 无关 C、无关 D、正确答案 other methods E、加强

2、28s  weaken
premise let plant to remove excess carbon dioxide
conclusion establish giant floating seaweed farms in the oceans
推测 burn seaweed will cause more carbon dioxide 答案与seaweed 和 carbon dioxide有关
选项分析 A、无关 B、正确 C、用不用 无关 D、扩大范围 E、赚钱 无关

3、41s  weaken
premise V only sold a few videos
conclusion C decline are not due to V that sell videos
推测 people who buy the videos will lend them to others
选项分析 A、无关比较 B、开新店 加强 C、无关信息D、无关 加强的感觉 E 、正确答案

4、33s weaken
premise  D is similar to M but D's label is gold color
conclusion do not buy D's wine whille thet really want to buy M's wine
推测  people will not see the label label 没有用 选项与 label 和customers有关
选项分析 A、无关 B、加强 C、无关D、正确答案 E 、加强

5、46s 归纳题
premise 30%的订阅者回应广告的是30岁以下的人     大多数回应广告的人是30岁以下的人
conclusion(推测) 30岁的订阅者很可能回应广告
选项分析 A?B、不能推出C、正确答案D、钱 无关E、正确答案 不理解
作者: sherryli    时间: 2012-4-26 16:57
Question 1:
Premise: little room for growth in carpet market because only few times for one person to buy carpet
Conclusion: Only through purchasing competitors can gain the market share

Weaken? If we can prove that it is not the only way to gain market share, it will weaken the conclusion.
A.    Irrelevant, we are not talking about other coverings.
B.    Irrelevant.
C.    Irrelevant. We are not talking about the profit.
D.    Correct. Price reduction can be another way to gain more market share.
E.    Irrelevant. Resistant to new patterns unrelated to the conclusion

Question 2:
Fact 1: Remove excess carbon dioxide by plant life
Fact 2:  one plan to establish the seaweed farms
Fact 3: Burned for fuel after seaweed plants die

If  we can prove that giant seaweed will not decrease the carbon dioxide, it will weaken the conclusion. Will the burning of seaweed plants release more carbon dioxide?
A.    Irrelevant
B.    Correct.
C.    Irrelevant.
D.    Strengthen
E.    Irrelevant

Question 3:
Jennifer:  10,000 fewer video rentals ---due to opening of V (sold videos cheaply)
Brad: sold 4,000 videos could not cause the decrease of 10,000 rentals

If we prove that sold 4,000 videos could cause the decrease of 10,000 rentals, it will weaken the explanation.
A.    Irrelevant
B.    Irrelevant
C.    Irrelevant
D.    Irrelevant
E.    Correct.

MWW new wine use similar bottle as ours—customers may mistake their wines to ours
DW: two bottle can be distinguished—because the label on ours is gold colored
                            If we prove that gold label cannot be a way to distinguish the wine
A.    Strengthen
B.    Irrelevant
C.    Irrelevant
D.    Correct
E.    Irrelevant

Regarding to Question 5, I was so confused. Help?
作者: zada2010    时间: 2012-4-26 20:47
2‘27 D 题干说只能通过purchasing competitors, weak D通过排挤出competitior一样可以获得更大的市场份额。
1'12 B
1’16E
1‘16D
2'37A
作者: FB小贝    时间: 2012-4-30 16:56
【1】Premise: the growth in the carpet market is tied to the size of the population and most who purchase carpet do so only once or twice
Conclusion: little room for the growth in the overall carpet market
Weaken: new customers will emerge as people age
a. irrelevant
b. irrelevant
c. strengthen
d. weaken, points out that there is aggressive marketing
e. irrelevant
【2】Premise: Establish giant floating seaweed farms in the oceans and dead seaweed plants will be disposed of by being burned for fuel
Conclusion: remove excess carbon dioxide from the atmosphere by increasing the amount that is absorbed by plant life
Weaken: being burned for fuel will emit carbon dioxide
a. irrelevant
b. correct
c. irrelevant ,kind of strengthen
d. irrelevant
e. irrelevant
【3】Premise: Videorama sold 4000 videos in 1994
Conclusion: the decline of 10000 is not caused by the new opening Videorama
Weaken: Videorama rent a lot of videos in 1994【people rent videos to each other】
a. not sufficient
b. irrelevant
c. irrelevant
d. irrelevant
e. correct
【4】Premise: the label is different
Conclusion: customers will not mistakenly buy theirs instead
Weaken: customers do not care about the label
a. irrelevant
b. strengthen
c. irrelevant
d. correct
e. irrelevant
【5】subscribers:30 precent – under 35
People:Most – under 35
a. irrelevant
b. irrelevant
c. irrelevant
d. irrelevant
e. correct
作者: dwindwin1106    时间: 2012-5-1 07:50
(1)P: the size of the carpet market is limited.
P:people generally purchase carpets once or twice, one in 20-30, the other 50-60
C:Company producing carpets will gain market share only through purchasing competitors and not through more aggressive market
Weaken: people are willing to purchase the carpets from the competitors
修正思路:手段目的题,还存在其他的手段使得能够达到目的
Answer: D
A.Irrelevant ------ the topic is carpet not other floor coverings
B.Strengthen ------ this choice shows that company will gain market shares only through the method mentioned in the stimuli because of no more niche for new brands remaing.
C.Irrelevant ------ the situations of the newly merged companies do nothing to weaken the argument.
D.Correct
E.Irrelevant ------ the habit of consumers does nothing to weaken the argument.

(2)Way: increase the CO2 amount that is absorbed by plant life. The plan is to establish seaweed farms in the ocean. They will be burned after their death.
Purpose: remove excess CO2 from the atmosphere
Weaken: the burning of seaweeds may create more CO2 than that absorbed by seaweeds
Answer: B
A.irrelevant
B.correct
C.irrelevant
D.irrelevant
E.irrelevant

(3)J: P: V opened in 1994 and is the first & only video rental outlets that also sell videos cheaply
           C: video rental outlets in C rent fewer videos in 1994 than in 1993 due to the opening of V
B: P: the decline (10000) > the videos sold out by V (4000)
   C: the decline was not cause by the opening of V
Weaken(B): V rent many videos in 1994
Answer: A
订正:E
A.Correct                 A. irrelevant
B.Irrelevant               B. irrelevant
C.Irrelevant               C. irrelevant
D.Irrelevant               D. irrelevant
E.Irrelevant               E. correct

(4)M: P: Our most popular wine is sold in a tall, black bottle. DW’s new wine has the similar bottle to ours
        C: customers who want to buy our wine will buy DW’s wine by mistake
  D: P: the label on DW’s bottle is gold, different from M’s bottle.
      C: customers will not be confused.
Weaken(DW): the label is too small for the customers to discern.
Answer: D
A.irrelevant
B.irrelevant
C.strengthen
D.correct
E.irrelevant

(5)MS:30% of the orders in response to the ad in the mag last year were placed by subscribers <35 year old.
   Ad: most of the orders placed in response to the ad in sys last year were placed by people <35 year old
Resolve: subscribers 不等于 people
Answer: E
A: irrelevant
B. irrelevant
C. conflict the second survey
D. irrelevant
作者: dwindwin1106    时间: 2012-5-1 08:09
关于逻辑链的第三题(video rental)有问题想请教一下,Brad的话中说videorama只sell了400,而其他的outlets确损失了10000,所以这个损失不能说是因为videorama的开张造成的。

题目是让weaken Brad的观点,我一开始是觉得如果如果Videorama租出的比卖出的多,那就能弥补这个当中购得差额了,所以一开始选了A。后来看了大家的解释后觉得E确实是一个更佳的答案。

那我想请问A为什么是错的呢?是因为它说借出的更多,到底多多少,是不是不够充分?还是说因为它没有针对题干中Brad的话提出质疑(题干只是说了卖掉的和减少的,并没有设计租出的量)呢?

可能问的比较罗嗦,呵呵,不好意思,希望能够得到回答,谢谢咯
作者: iaoinging    时间: 2012-5-6 02:14
第一篇:1. 36
       2. Background: relative little room for carpet market because of the size of the population.
          Conclusion: companies should through purchasing competitors to gain market share, not through more aggressive marketing.
       3. what if there are less companies.
       4. C
 A,  我觉得是无关选项。题目问的是gain market share in the carpet market.
 B,  这个选项其实应该是支持文中结论吧。因为市场是小的,那么只能通过并购来提高市场占有量。哦,这个只是说明一个事实。并没有说他不成功的原因/
 C, 说明的一个事实,就75%的合并都是不成功的。应该是这个吧。
 D, 这个应该也是支持,说明merger不是唯一的方法。应该选这个的!
 E, 无关选项吧。

第二篇:1. 25s
       2. scientists want to remove EXCESS carbon dioxide by increasing the amount that is absorbed plants. and this plan is establish giant floating seaweed, after the seaweeds die, they will be disposed of by being burned for fuel.
       3. what if those seaweed will absorbed more than excess carbon dioxide? What if the seaweed will product more carbon dioxide after they die?
       4. B
 A, 无关选项。题目并没有问实行这个计划的可行性。而是问如果这个计划可以实施,会产生什么样的不好处。
 B, 如果这种seaweeds 死后会产生更多CD的话,那么这个计划就是徒劳无功的。
 C, 无关选项,这已经是之后要考虑的问题。
 D, 这是提供一个事实,证明CD太多了,植物吸收不来。
 E, 题目并没有问道cost的问题。所以应该也是无关。

第三篇:1. 47s
       2. because  V sold only 4,000 videos in 1994, while C lost 10,000 orders of rental. So that it not because V.
       3. B的假设是建立在一个rental的人会买一张video。 可是也许,一个customer 买了一个video后,原本要租的他身边的朋友都不用去租了的话。那么J说的结果是有可能的。
       4. E
A, 这应该是一个track的选项, 可是B对比的是V卖出去的和C失去的租界订单,所以V租出去的应该是没有关系的。
B, 题中并没有提到,无关选项。
C, 无关
D, 无关
E, 说的多好呀,还不准我买了之后借给我朋友啦。



第四篇:1. 28S
       2. MW stated that DW product a new wine with a similar design on bottle. Therefore, MW’s customers will mistakenly buy DW’s product.
         While DW said since the label is different so that the claim from MW is not right.
       3. what if the label is really small that no one will pay attention to? What if MW’s label also has a gold color?
       4. D
A, 无关选项。DW的label的颜色没有什么可考察性。
B,这个是支持DW的说法的。
C,无关选项
D,一般客户都是根据general impression去选择酒的。削弱了DW的说法。
E, 好像是支持DW的说法的。

第五题, 1. 35s
        2. first finding, subscribers里订购M的人有30% 的是小于35 岁的subscribers.
          Second finding, 订购M的人有35% 是小于35岁的。
        3. 我最讨厌这种题了。所以不看选项完全没思路。不过估计也就是注意总的订购M的人数和订购M里subscribers人数的区别了。
        4. C
A, 这个好像是相反的
B, 这个是无关选项
C, 既然只有百分之30是小于三十的,那么大部分应该都是大于三十的。
D, 这个是相反的
E, 无关啦。
作者: iaoinging    时间: 2012-5-6 02:17
我认为哦,不晓得是不是对的。
B
的假设是建立在一个rental的人会买一张video可是也许,一个customer 买了一个video后,原本要租的他身边的朋友都不用去租了的话。那么J说的结果是有可能的。A说的是V租出去的可是B对比的是V卖出去的和C失去的租界订单,所以V租出去的应该是没有关系的。
可是B对比的是V卖出去的和C失去的租界订单,所以V租出去的应该是没有关系的。
作者: 纸鸢飞离    时间: 2012-5-7 17:19
读题:35''



逻辑链:background informationthere is little room to expend the carpet market



       remisemost people only buy carpets twice in their lives.



       Conclusionso companies only can gain market by purchasing their competitor's carpet, but not by aggressive marketing.



Weaken



Other aggressive marketing also could help to gain market share.
(A) Most of the major carpet producers market
other floor coverings as well.(irrelevant)
(B) Most established carpet producers market
several different brand names and varieties,
and there is no remaining niche in the
market for new brands to fill.(support)
(C) Two of the three mergers in the industrys last
ten years led to a decline in profits and
revenues for the newly merged companies.(irrelevant)
(D) Price reductions, achieved by cost-cutting in
production, by some of the dominant firms
in the carpet market are causing other
producers to leave the market altogether.(other merger method also could gain more market)
(E) The carpet market is unlike most markets in
that consumers are becoming increasingly
resistant to new patterns and styles.(support)
2.
Scientists are discussing ways to remove excess carbon dioxide from the atmosphere by increasing the amount that is absorbed by plant life. One plan to accomplish this is to establish giant floating seaweed farms in the oceans. When the seaweed plants die, they will be disposed of by being burned for fuel.
Which of the following, if true, would indicate the most serious weakness in the plan above?
-- PREP07 CR1 NO.9



读题:



逻辑链:background information:scientist plan to remove the Co2 by plant more plants to absorb the excess Co2.



       remise:they plan to plant floating seaweed farm in the oceans.



       Conclusion: it not only could absorb the Co2, but also be burnt to be fuel after its death.



Weaken: it will produce more CO2 when it being burnt.

(A) Some areas of ocean in the Southern Hemisphere do not contain sufficient nutrients to support large seaweed farms.(irrelevant, can not refute the premise)
(B) When a seaweed plant is burned, it releases an amount of carbon dioxide comparable to the amount it has absorbed in its lifetime.(right,because it setoff the contribution it made to remove the CO2.)
(C) Even if seaweed farms prove effective, some people will be reluctant to switch to this new fuel.(not the direct weakness)
(D) Each year about seven billion tons of carbon dioxide are released into the atmosphere but only about five billion tons are absorbed by plant life.(cannot weaken)
(E) Seaweed farms would make more money by farming seaweed to sell as nutritional supplements than by farming seaweed to sell as fuel.(irrelevant)


3.
Jennifer: Video rental outlets in Centerville together handled 10,000 fewer video rentals in 1994 than in 1993. The decline in rentals was probably due almost entirely to the February 1994 opening of Videorama, the first and only video rental outlet in the area that, in addition to renting videos, also sold them cheaply.

Brad: There must be another explanation: as you yourself said, the decline was on the order of 10,000 rentals. Yet Videorama sold only 4,000 videos in 1994.

Which of the following, if true, would most seriously weaken the force of the objection that Brad presents to Jennifer's explanation?
-- PREP07 CR1 NO.10



读题:29.6''



逻辑链:background information:Video rental outlets in Centerville together handled 10,000 fewer video rentals in 1994 than in 1993.



       remise:f



       Conclusion:Videorama, the first and only video rental outlet in the area that, in addition to renting videos, also sold them cheaply



Weaken:in 1994, there are more than 4000 videos.

(A) In 1994 Videorama rented out more videos than it sold.
(B) In 1994 two new outlets that rent but that do not sell videos opened in Centerville.(irrelevant)
(C) Most of the video rental outlets in Centerville rent videos at a discount on certain nights of the week.(irrelevant)
(D) People often buy videos of movies that they have previously seen in a theater.



(E) People who own videos frequently loan them to their friends.

4.
Mourdet Winery: Danville Winery's new wine was introduced to compete with our most popular wine, which is sold in a distinctive tall, black bottle. Danville uses a similar bottle. Thus, it is likely that many customers intending to buy our wine will mistakenly buy theirs instead.

Danville Winery: Not so. The two bottles can be readily distinguished: the label on ours, but not on theirs, is gold colored.

Which of the following, if true, most undermines Danville Winery's response?
-- PREP07 CR1 NO.11



读题:51''



逻辑链:background information will produce a wine to compete with our wine,because our bottle is bigger than theirs, customers will not buy ours.



       remise:because there are some differences in appearance on the bottle, so may be customers will not buy ours.



       Conclusion:so customers will still buy ours mistakenly.



Weaken:Although the customers distinguish the difference, it will do nothing about our sales.




(A) Gold is the background color on the label of many of the wines produced by Danville Winery.(irrelevant
(B) When the bottles are viewed side by side, Danville Winery's bottle is perceptibly taller than Mourdet Winery's.(irrelevant )
(C) Danville Winery, unlike Mourdet Winery, displays its wine's label prominently in advertisements.support
(D) It is common for occasional purchasers to buy a bottle of wine on the basis of a general impression of the most obvious feature of the bottle.irrelevant
(E) Many popular wines are sold in bottles of a standard design.
是说形状的差异比label的差异更引人瞩目吗?

5.
Finding of a survey of Systems magazine subscribers: Thirty percent of all merchandise orders placed by subscribers in response to advertisements in the magazine last year were placed by subscribers under age thirty-five.

Finding of a survey of advertisers in Systems magazine: Most of the merchandise orders placed in response to advertisements in Systems last year were placed by people under age thirty-five.

For both of the findings to be accurate, which of the following must be true?
-- PREP07 CR1 NO.12



读题:43''



逻辑链:



Background information



Premise30%percent of all are placed by people under 35.



         Most of placed are people under35.































Conclusion



Infer:



(A) More subscribers to Systems who have never ordered merchandise in response to advertisements in the magazine are age thirty-five or over than are under age thirty-five.
(B) Among subscribers to Systems, the proportion who are under age thirty-five was considerably lower last year than it is now. Irrelevant
(C) Most merchandise orders placed in response to advertisements in Systems last year were placed by Systems subscribers over age thirty-five. opposite
(D) Last year, the average dollar amount of merchandise orders placed was less for subscribers under age thirty-five than for those age thirty-five or over. irrelevant
E) Last year many people who placed orders for merchandise in response to advertisements in Systems were not subscribers to the magazine.



不懂




求解答啊求解答


2.
Scientists are discussing ways to remove excess carbon dioxide from the atmosphere by increasing the amount that is absorbed by plant life. One plan to accomplish this is to establish giant floating seaweed farms in the oceans. When the seaweed plants die, they will be disposed of by being burned for fuel.
Which of the following, if true, would indicate the most serious weakness in the plan above?
-- PREP07 CR1 NO.9



读题:



逻辑链:background information:scientist plan to remove the Co2 by plant more plants to absorb the excess Co2.



       remise:they plan to plant floating seaweed farm in the oceans.



       Conclusion: it not only could absorb the Co2, but also be burnt to be fuel after its death.



Weaken: it will produce more CO2 when it being burnt.

(A) Some areas of ocean in the Southern Hemisphere do not contain sufficient nutrients to support large seaweed farms.(irrelevant, can not refute the premise)
(B) When a seaweed plant is burned, it releases an amount of carbon dioxide comparable to the amount it has absorbed in its lifetime.(right,because it setoff the contribution it made to remove the CO2.)
(C) Even if seaweed farms prove effective, some people will be reluctant to switch to this new fuel.(not the direct weakness)
(D) Each year about seven billion tons of carbon dioxide are released into the atmosphere but only about five billion tons are absorbed by plant life.(cannot weaken)
(E) Seaweed farms would make more money by farming seaweed to sell as nutritional supplements than by farming seaweed to sell as fuel.(irrelevant)


3.
Jennifer: Video rental outlets in Centerville together handled 10,000 fewer video rentals in 1994 than in 1993. The decline in rentals was probably due almost entirely to the February 1994 opening of Videorama, the first and only video rental outlet in the area that, in addition to renting videos, also sold them cheaply.

Brad: There must be another explanation: as you yourself said, the decline was on the order of 10,000 rentals. Yet Videorama sold only 4,000 videos in 1994.

Which of the following, if true, would most seriously weaken the force of the objection that Brad presents to Jennifer's explanation?
-- PREP07 CR1 NO.10



读题:29.6''



逻辑链:background information:Video rental outlets in Centerville together handled 10,000 fewer video rentals in 1994 than in 1993.



       remise:f



       Conclusion:Videorama, the first and only video rental outlet in the area that, in addition to renting videos, also sold them cheaply



Weaken:in 1994, there are more than 4000 videos.

(A) In 1994 Videorama rented out more videos than it sold.
(B) In 1994 two new outlets that rent but that do not sell videos opened in Centerville.(irrelevant)
(C) Most of the video rental outlets in Centerville rent videos at a discount on certain nights of the week.(irrelevant)
(D) People often buy videos of movies that they have previously seen in a theater.



(E) People who own videos frequently loan them to their friends.

4.
Mourdet Winery: Danville Winery's new wine was introduced to compete with our most popular wine, which is sold in a distinctive tall, black bottle. Danville uses a similar bottle. Thus, it is likely that many customers intending to buy our wine will mistakenly buy theirs instead.

Danville Winery: Not so. The two bottles can be readily distinguished: the label on ours, but not on theirs, is gold colored.

Which of the following, if true, most undermines Danville Winery's response?
-- PREP07 CR1 NO.11



读题:51''



逻辑链:background information will produce a wine to compete with our wine,because our bottle is bigger than theirs, customers will not buy ours.



       remise:because there are some differences in appearance on the bottle, so may be customers will not buy ours.



       Conclusion:so customers will still buy ours mistakenly.



Weaken:Although the customers distinguish the difference, it will do nothing about our sales.




(A) Gold is the background color on the label of many of the wines produced by Danville Winery.(irrelevant
(B) When the bottles are viewed side by side, Danville Winery's bottle is perceptibly taller than Mourdet Winery's.(irrelevant )
(C) Danville Winery, unlike Mourdet Winery, displays its wine's label prominently in advertisements.support
(D) It is common for occasional purchasers to buy a bottle of wine on the basis of a general impression of the most obvious feature of the bottle.irrelevant
(E) Many popular wines are sold in bottles of a standard design.
是说形状的差异比label的差异更引人瞩目吗?

5.
Finding of a survey of Systems magazine subscribers: Thirty percent of all merchandise orders placed by subscribers in response to advertisements in the magazine last year were placed by subscribers under age thirty-five.

Finding of a survey of advertisers in Systems magazine: Most of the merchandise orders placed in response to advertisements in Systems last year were placed by people under age thirty-five.

For both of the findings to be accurate, which of the following must be true?
-- PREP07 CR1 NO.12



读题:43''



逻辑链:



Background information



Premise30%percent of all are placed by people under 35.



         Most of placed are people under35.































Conclusion



Infer:



(A) More subscribers to Systems who have never ordered merchandise in response to advertisements in the magazine are age thirty-five or over than are under age thirty-five.
(B) Among subscribers to Systems, the proportion who are under age thirty-five was considerably lower last year than it is now. Irrelevant
(C) Most merchandise orders placed in response to advertisements in Systems last year were placed by Systems subscribers over age thirty-five. opposite
(D) Last year, the average dollar amount of merchandise orders placed was less for subscribers under age thirty-five than for those age thirty-five or over. irrelevant
E) Last year many people who placed orders for merchandise in response to advertisements in Systems were not subscribers to the magazine.



不懂




求解答啊求解答


2.
Scientists are discussing ways to remove excess carbon dioxide from the atmosphere by increasing the amount that is absorbed by plant life. One plan to accomplish this is to establish giant floating seaweed farms in the oceans. When the seaweed plants die, they will be disposed of by being burned for fuel.
Which of the following, if true, would indicate the most serious weakness in the plan above?
-- PREP07 CR1 NO.9



读题:



逻辑链:background information:scientist plan to remove the Co2 by plant more plants to absorb the excess Co2.



       remise:they plan to plant floating seaweed farm in the oceans.



       Conclusion: it not only could absorb the Co2, but also be burnt to be fuel after its death.



Weaken: it will produce more CO2 when it being burnt.

(A) Some areas of ocean in the Southern Hemisphere do not contain sufficient nutrients to support large seaweed farms.(irrelevant, can not refute the premise)
(B) When a seaweed plant is burned, it releases an amount of carbon dioxide comparable to the amount it has absorbed in its lifetime.(right,because it setoff the contribution it made to remove the CO2.)
(C) Even if seaweed farms prove effective, some people will be reluctant to switch to this new fuel.(not the direct weakness)
(D) Each year about seven billion tons of carbon dioxide are released into the atmosphere but only about five billion tons are absorbed by plant life.(cannot weaken)
(E) Seaweed farms would make more money by farming seaweed to sell as nutritional supplements than by farming seaweed to sell as fuel.(irrelevant)


3.
Jennifer: Video rental outlets in Centerville together handled 10,000 fewer video rentals in 1994 than in 1993. The decline in rentals was probably due almost entirely to the February 1994 opening of Videorama, the first and only video rental outlet in the area that, in addition to renting videos, also sold them cheaply.

Brad: There must be another explanation: as you yourself said, the decline was on the order of 10,000 rentals. Yet Videorama sold only 4,000 videos in 1994.

Which of the following, if true, would most seriously weaken the force of the objection that Brad presents to Jennifer's explanation?
-- PREP07 CR1 NO.10



读题:29.6''



逻辑链:background information:Video rental outlets in Centerville together handled 10,000 fewer video rentals in 1994 than in 1993.



       remise:f



       Conclusion:Videorama, the first and only video rental outlet in the area that, in addition to renting videos, also sold them cheaply



Weaken:in 1994, there are more than 4000 videos.

(A) In 1994 Videorama rented out more videos than it sold.
(B) In 1994 two new outlets that rent but that do not sell videos opened in Centerville.(irrelevant)
(C) Most of the video rental outlets in Centerville rent videos at a discount on certain nights of the week.(irrelevant)
(D) People often buy videos of movies that they have previously seen in a theater.



(E) People who own videos frequently loan them to their friends.

4.
Mourdet Winery: Danville Winery's new wine was introduced to compete with our most popular wine, which is sold in a distinctive tall, black bottle. Danville uses a similar bottle. Thus, it is likely that many customers intending to buy our wine will mistakenly buy theirs instead.

Danville Winery: Not so. The two bottles can be readily distinguished: the label on ours, but not on theirs, is gold colored.

Which of the following, if true, most undermines Danville Winery's response?
-- PREP07 CR1 NO.11



读题:51''



逻辑链:background information will produce a wine to compete with our wine,because our bottle is bigger than theirs, customers will not buy ours.



       remise:because there are some differences in appearance on the bottle, so may be customers will not buy ours.



       Conclusion:so customers will still buy ours mistakenly.



Weaken:Although the customers distinguish the difference, it will do nothing about our sales.




(A) Gold is the background color on the label of many of the wines produced by Danville Winery.(irrelevant
(B) When the bottles are viewed side by side, Danville Winery's bottle is perceptibly taller than Mourdet Winery's.(irrelevant )
(C) Danville Winery, unlike Mourdet Winery, displays its wine's label prominently in advertisements.support
(D) It is common for occasional purchasers to buy a bottle of wine on the basis of a general impression of the most obvious feature of the bottle.irrelevant
(E) Many popular wines are sold in bottles of a standard design.
是说形状的差异比label的差异更引人瞩目吗?

5.
Finding of a survey of Systems magazine subscribers: Thirty percent of all merchandise orders placed by subscribers in response to advertisements in the magazine last year were placed by subscribers under age thirty-five.

Finding of a survey of advertisers in Systems magazine: Most of the merchandise orders placed in response to advertisements in Systems last year were placed by people under age thirty-five.

For both of the findings to be accurate, which of the following must be true?
-- PREP07 CR1 NO.12



读题:43''



逻辑链:



Background information



Premise30%percent of all are placed by people under 35.



         Most of placed are people under35.































Conclusion



Infer:



(A) More subscribers to Systems who have never ordered merchandise in response to advertisements in the magazine are age thirty-five or over than are under age thirty-five.
(B) Among subscribers to Systems, the proportion who are under age thirty-five was considerably lower last year than it is now. Irrelevant
(C) Most merchandise orders placed in response to advertisements in Systems last year were placed by Systems subscribers over age thirty-five. opposite
(D) Last year, the average dollar amount of merchandise orders placed was less for subscribers under age thirty-five than for those age thirty-five or over. irrelevant
E) Last year many people who placed orders for merchandise in response to advertisements in Systems were not subscribers to the magazine.



不懂




求解答啊求解答
作者: 卖红薯a    时间: 2012-5-11 21:53
1.
1)计时:44s
2)逻辑链Situation
Background Information: The growth of carpet market which is tied to the size of population is hard to grow.
Premise: Most consumers buy carpet twice, which one in their 20s or 30s and one if their 50s or 60s.
Conclusion: Carpet producers are able to gain market share not through more aggressive marketing but purchasing competitors.
3)推测:Most consumers buy twice ≠ they won’t buy the third time, even more
Gain from purchasing competitors may not be easier than from aggressive markets
4)分析选项:选E
A.Irrelevant. 没看懂
B.Opposite.
C.Irrelevant.
D.Irrelevant. mainly talking about the other producers
E.Right.


2.
1)计时:34s
2)逻辑链Situation
Background Information: There’s a way to remove excess carbon dioxide by increasing the amount that is absorbed by plants.
Premise: The base of this plan is establishing giant floating seaweed farms in the sea.
Conclusion: The seaweed plants will be disposed of by being burned for fuel after they die.
3)推测: The carbon dioxide will still be back into the atmosphere when the seaweed plants are burned.
4)分析选项:选B
A.Irrelevant.
B.Right.
C.Irrelevant. the subject is people
D.Opposite. 不确定
E.Irrelevant.


3.
1)计时:50s
2)逻辑链Situation
Background information: There’s a 10,000 decrease of rental videos in Centerville.
Premise: The decrease was probably due to the opening of Videorama which including rent and cheaply sell videos.
Conclusion: The decrease was on rental not on selling.
3)推测: Video rental outlets in Centerville maybe sold more than it had done before.
Videorama rent big, but it didn’t sell big.
4)分析选项:选B
A.Irrelevant.
B.Right.
C.Opposite.
D.Irrelevant.
E.Irrelevant.


4.
1)计时:43s
2)逻辑链Situation
Background information: The bottle of DW’s new wine is similar to ours.
Premise: Customers who want to buy our wine might be misguided.
Conclusion: DW said:” The label color is gold which is different from theirs.”
3)推测: Consumers can easily consider the new gold label as MW’s new products.
4)分析选项:选D
A.Opposite.
B.Opposite.
C.Irrelevant.
D.Right.
E.Opposite.


5.
1)计时:60s
2)逻辑链Situation
Background information:
Magazine subscribers: 35% orders were placed by subscribers under age 35.
Advertisers: most orders were placed by people under age 35.
3)分析选项:选D
A.Opposite.
B.Irrelevant.
C.Opposite.
D.Right.
E.Irrelevant. 没看懂
作者: spencerX    时间: 2012-5-12 18:34
补上昨天的...

1.D
P:
There is relatively little room for carpet company to increase their total profit because the size of population. Most people will buy carpet only twice in their lifetime.
C:
As the population ages, companies can only increase their market share through purchasing competitors rather than marketing.
Pre:
There are other ways to increase market share.
A:
A. Irrelevant. What take about in the argument is carpet, not other floor coverings.
B. If there is no remaining niche in the market for new brands, maybe companies can increase their market share only by purchasing competitors.
C. Shell game. Profit≠market share
D. Correct. If other producers leave the market, then the market share of company will increase, so there is no need to purchase other competitors.
E. If customers don’t want new style, then whatever the company do will not increase the market share.
------------------------------------------------------------------------------------------------------------------------------------------------------------
2. B
B:
Scientists try to decrease the carbon dioxide through increase the amount absorbed by plants.
P:
When the seaweed plants die, they will be disposed of by being burned for fuel.
C:
Excessive carbon emission will be reduced through establishing giant floating seaweed farms in the oceans.
Pre:
Plants may release more co2 during their burning than their absorbed.
A. Even if some areas of ocean in the Southern Hemisphere do not contain sufficient nutrients to support large seaweed farms, there will be some places can. So it cannot weaken the argument.
B. Correct! If this situation is real, then the strategy taken by scientists cannot reduce the amount of carbon dioxide.(对于方法可行性的质疑)
C. Although “some” people will be reluctant to switch to new fuel, this is not relevant to what we talk about because we just want to assess whether the strategy itself can be useful.
D. This answer proves that the method taken by scientist can be useful at some extent, so it cannot indicate the most serious weakness.
E. Irrelevant
------------------------------------------------------------------------------------------------------------------------------------------------------------
3. E
Jennifer:
B:
Video rental have decreased from 1994 to 1993.
P
The first and only video rental outlet in the area both rented videos and sold them cheaply
C:
The decline in rentals was probably due almost entirely to opening of Videorama
Brad:
P: sales< the decline
C: There is another reason account for the decrease of rent.
Pre: 卖出去的video被许多人传看,或者大家一起看?
A. To some extent, this choice slightly support the argument made by Brad.
B. What talked in this choice is out of scope.
C. Irrelevant.
D. Irrelevant
E. Correct. If people who buy videos often rent them to friends, then some people will neither have to purchase videos nor rent videos in the rental outlet.
------------------------------------------------------------------------------------------------------------------------------------------------------------
4. D
MW:
P:DW出售的酒的瓶子和MW最popular非常类似。
C: 原来想买MW酒的人可能会错买DW
DW:
PW的酒瓶和MW的酒瓶有细微的差别。
C:The two bottles can be distinguished.
Pre: The slight difference cannot be notice.

A. Irrelevant
B. This actually support the argument. If people can notice the difference, then the claim that the two bottles can be readily distinguished is right.
C. Irrelevant
D. We know that the difference between the bottles made by two people is slight, so people who buy a bottle of wine on the basis of a general impression cannot tell two kind of bottles apart.
E. Irrelevant
------------------------------------------------------------------------------------------------------------------------------------------------------------

5. C
FACT: 1.回应广告的subscribers有30%是被35岁以下的subscribers取代的。
    2.大多数回应广告的人是被三十五岁以下的人取代的。
错误原因:没有看懂题目,完全是乱猜的。搞不清范围。

做精练得到的收获:
To attack a conditional statement you must show that the necessary condition is not actually necessary for the sufficient condition to occur.
充要条件的weaken方法)

作者: Shirley55555    时间: 2012-5-16 10:10
1 1'11'' P: people seldom buy carpet during their life     C: company could enlarge their market share through purchasing competitors otjan than marketing.
           W: Company could enlarge their market share by through marketing.eg. marketing persuade  ppl to buy.   Choose C   ABD irr E support  
           正确答案:D 错误原因:1是没记清楚逻辑链 没有区分profits 和 marketshare 2没意识到降价也是marketing 以为marketing就是纯粹推销

2  32'' P: plants absorb carbon dioxide C: plant seaweed on the ocean could solve the too much CD problem because seaweed die while burn the CD.
          W: seaweed produce CD when they alive or sth like that.   Choose B ACDE irr

3 1'03'' P rental decline 10000 because V open. C: the assumption is wrong because V sold only 4000.  
          W: V sold 4000 but rent more.   Choose A     BCDE irr
          正确答案:E引入他因 A不充分 more没说多少 不见得达到1万 或者题目中的rental是指整个地区,A说more 反对条件了?

4  30''  : D use the similar bottle as ours, ppl would be mistaken  C: our lable is different so ppl could distinguish them:  
          W: ppl notice the bottle more than lable   Choose D   AB support  CE irr

5  51'' P: 1.>30% order under 35 . 2. most order under 35.
         True: most order below 35.     Choose A BCDE irr
         正确答案:E 当时没看懂题。。。 正确的理解应该是1 30% m order S of system is under 35 2 most of m order S is under 35 推出most s id not belong to s of system...
作者: baseboss    时间: 2012-5-16 16:25
B: carpet market tied to the size of population
P: Most people only buy once or twice in the whole life.
D: carpet company want to more competitiveness only through buy

competitor,not throught aggressive way.

推测:
1. people will buy more than 3 times in whole life due to

marriage,remarriage or buy a new house.
2. people want to change from traditional carpet to art carpet.

(A) Most of the major carpet producers market
other floor coverings as well.
irrevelent -->about floor coverings

(B) Most established carpet producers market
several different brand names and varieties,
and there is no remaining niche in the
market for new brands to fill.
support not weaken

(C) Two of the three mergers in the industry’s last
ten years led to a decline in profits and
revenues for the newly merged companies.
irrevelent

(D) Price reductions, achieved by cost-cutting in
production, by some of the dominant firms
in the carpet market are causing other
producers to leave the market altogether.
right.

(E) The carpet market is unlike most markets in
that consumers are becoming increasingly
resistant to new patterns and styles.
irrevelent


2
B:reduce co2
p:build plant to absorb c02
c:build giant seawood on the oceans,and after seawood died can be

burn for feul.
推测;
burn seawood will increase co2.
选B

3.
B:video rent decline due to Videorama
P:video rent decline 10000 but videorama only 4000
c:video rent devline not due to videorama.
推测:
1.Due to videorama people do not want to rent video.
2.due to videorama people think video out date.
选E

4.
B:wine compete on the market
P: competitor use the same bottle will lead customer mis-distinguish
c:customer will not mis-distinguish due to lab is difference.
推测:
Normally,people will notice the bottle rather than lab.
选D

5
B:suvey on the subscribes of magazine and on the advertisement of the magazine
P: 30% subscribes order magazine due to they want to advertise on the magazine.and most of subscribes order the magazine due to they want to advertise.
c: NA
推测:
age under 35 do not want to order magazine.
people only want to advertise on the magazine ,not want to read the magazine under age 35
选E
作者: baseboss    时间: 2012-5-16 16:25
B: carpet market tied to the size of population
P: Most people only buy once or twice in the whole life.
D: carpet company want to more competitiveness only through buy

competitor,not throught aggressive way.

推测:
1. people will buy more than 3 times in whole life due to

marriage,remarriage or buy a new house.
2. people want to change from traditional carpet to art carpet.

(A) Most of the major carpet producers market
other floor coverings as well.
irrevelent -->about floor coverings

(B) Most established carpet producers market
several different brand names and varieties,
and there is no remaining niche in the
market for new brands to fill.
support not weaken

(C) Two of the three mergers in the industry’s last
ten years led to a decline in profits and
revenues for the newly merged companies.
irrevelent

(D) Price reductions, achieved by cost-cutting in
production, by some of the dominant firms
in the carpet market are causing other
producers to leave the market altogether.
right.

(E) The carpet market is unlike most markets in
that consumers are becoming increasingly
resistant to new patterns and styles.
irrevelent


2
B:reduce co2
p:build plant to absorb c02
c:build giant seawood on the oceans,and after seawood died can be

burn for feul.
推测;
burn seawood will increase co2.
选B

3.
B:video rent decline due to Videorama
P:video rent decline 10000 but videorama only 4000
c:video rent devline not due to videorama.
推测:
1.Due to videorama people do not want to rent video.
2.due to videorama people think video out date.
选E

4.
B:wine compete on the market
P: competitor use the same bottle will lead customer mis-distinguish
c:customer will not mis-distinguish due to lab is difference.
推测:
Normally,people will notice the bottle rather than lab.
选D

5
B:suvey on the subscribes of magazine and on the advertisement of the magazine
P: 30% subscribes order magazine due to they want to advertise on the magazine.and most of subscribes order the magazine due to they want to advertise.
c: NA
推测:
age under 35 do not want to order magazine.
people only want to advertise on the magazine ,not want to read the magazine under age 35
选E
作者: 二楼往下掉    时间: 2012-5-28 00:44
1.
(1)计时45s
(2)逻辑链
Primes
<1> There is relatively little room for growth in the overall carpet market, which is tied to the size of the population.
<2> Most who purchase carpet do so only once or twice, first in their twenties or thirties, and then perhaps again in their fifties or sixties.
Conclusion:
As the population ages, companies producing carpet will be able to gain market share in the carpet market only through purchasing competitors, and not through more aggressive marketing.
(3)猜测
<1> 有很多人移民到这个地方
<2> 这是一个旅游城市,有很多游客会购买,也就是说当地居民并不是唯一的消费者
我的答案:C
正确答案:D
今天实在是不清醒了,明儿来好好分析下。。。

2.
(1)计时45s
(2)逻辑链
plan:
<1> Establish giant floating seaweed farms in the oceans.
<2> when the seaweed plants die, they will be disposed of by being burned for fuel
Goal:
Remove excess carbon dioxide from the atmosphere
(3)猜测
<1> plants的燃烧会不会又产生carbon dioxide

3.
(1)计时:45s
(2)逻辑链
Jennifer:
Background:
Video rental outlets in Centerville together handled 10,000 fewer video rentals in 1994 than in 1993.
Primes:
The first and only video rental outlet in the area that, in addition to renting videos, also sold them cheaply
Conclusion:
The decline in rentals was probably due almost entirely to the February 1994 opening of Videorama
Brad:
Primes:
The decline was on the order of 10,000 rentals. Yet Videorama sold only 4,000 videos in 1994.
Conclusion:
There must be another explanation.
(3)猜测
<1> 人们互相借,而不从租借公司租借,所以虽然只卖了4000,但是租借减少了10000
<2> 有竞争

4.
(1)计时:45s
(2)逻辑链
MW:
Primes:
DW’s new wine ha the similar bottle with the MW’s.
conclusion:
It is likely that many customers intending to buy our wine will mistakenly buy theirs instead
DW:
Primes:
The label on DW is gold colored, but not the MW’s
Conclusion:
The two bottles can be readily distinguished
(3)猜测
<1> 商标太小看不见
<2> 很多人是通过瓶子来识别MW的,认为瓶子是MW的标志

5.
(1)计时:好久好久都没看懂
(2)由于没看懂,所以翻译了一下,不过貌似翻译的也不太对。。。
翻译:
对于订阅者的调查:30%的商品订单是由35岁以下的杂志订阅者提出的,这些订单是回应去年的广告的
对于广告者的调查:大多的订单是响应去年广告订单来自于35岁以下的人
我的答案:C
正确答案:E
没看懂题目,有待认真分析!!!
作者: adriana2011    时间: 2012-5-29 22:14
1)38S
2)premise:1) Most who purchase carpet do so only
once or twice;——There is relatively little room for growth in the
overall carpet market2
  Conclusion: gain market share ——〉through purchasing competitors, and not
through more aggressive marketing.
3)选:D 在DE之间犹豫了半天
A: Most of the major carpet producers还干点别的什么market相关的事情,无关
B:有support原文前提的感觉
C: last ten years 时间范围错,而且原文说的是market share
E: carpet market的一些特点 irrelevant


【逻辑链】
2.
Scientists are discussing ways to remove excess carbon dioxide from the atmosphere by increasing the amount that is absorbed by plant life.  One plan to accomplish this is to establish giant floating seaweed farms in the oceans.  When the seaweed plants die, they will be disposed of by being burned for fuel.
Which of the following, if true, would indicate the most serious weakness in the plan above?
-- PREP07 CR1 NO.9
1) 46s
2) establish giant floating seaweed farms in the oceans is a effective plan——〉remove excess carbon dioxide from the atmosphere
additional information:seaweed plants die, disposed of by being burned for fuel
3)选:B
A: Some areas of ocean in the Southern Hemisphere 无关
B:该方法在reduce Carbon dioxide的同时,又有可能导致carbon dioxide增加
C:some people will be reluctant to无关
D:一个关于plant life absorbed carbon dioxide的数据,至少说明该方法还是有效的,无法weaken结论
E:farming哪种东西make more money,无关

3.
1)38s
2) J: because Videorama (sold/rent)——video rents reduced 10000 from 1993 to 1994 in Centerville
  B: V-outlets sales only 4000——there must other reasons
3) 选A
A: rent多,所以说明可能是sales 4000不能说明实质问题
B: two new outlets. 超出范围
C:V-outlests 采取的促销措施,irrelevant
D/E:people的行为习惯,无关

4.
1)37s
2) MW: DW new wine, similar bottle with us——customer would mistaken
DW: not so : gold colors label is different
3)key: D
(A) irrelevant
(B)  a little strenthen
(C) irrelevant
(D) golden lable can not let consumer differentiate the two brands. Correct choice
(E)irrelevant


5.
1)50s
2) 晕晕
3)
(A) never ordered merchandise  -原文无
(B) Among subscribers to Systems, the proportion …lower last year than it is now.-原文无此比较
(C)  与原文信息相反
(D) the average dollar amount  原文无
作者: isa0709    时间: 2012-5-30 22:31
3. There is relatively little room for growth in the
overall carpet market, which is tied to the size of the
population. Most who purchase carpet do so only
once or twice, first in their twenties or thirties, and
then perhaps again in their fifties or sixties. Thus as
the population ages, companies producing carpet
will be able to gain market share in the carpet market
only through purchasing competitors, and not
through more aggressive marketing.
Which one of the following, if true, casts the most
doubt on the conclusion above?
--BIBLE WEAKEN NO.3
(A) Most of the major carpet producers market
other floor coverings as well.                          没有削断逻辑链
(B) Most established carpet producers market
several different brand names and varieties,
and there is no remaining niche in the
market for new brands to fill.                        某种程度上是支持非削弱
(C) Two of the three mergers in the industry’s last
ten years led to a decline in profits and
revenues for the newly merged companies.         某种程度上是支持,加强竞争的白热化
(D) Price reductions, achieved by cost-cutting in
production, by some of the dominant firms
in the carpet market are causing other
producers to leave the market altogether.           竞争者减少,与结论方向相反,削弱,注意此      
选项里有隐含的逻辑链
(E) The carpet market is unlike most markets in
that consumers are becoming increasingly
resistant to new patterns and styles.                没看到最后一行,只看到consumers  
Increasingly, 此选项既不削弱也不支持
计时:1.26
Premise: people purchase carpet only once or twice
Conclusion: people ages, carpet producer gain market share only through purchasing competitors not through more aggressive marketing.
答案预测: there are other methods to gain market share other than purchasing competitors. for example, new category to age people or push frequent generation change on carpet
E/D
作者: emmasy    时间: 2012-6-2 12:39
1.
premise:there is little room for the growth in the carpet market, since most consumers buy once or twice in their whole lives.
conclusion:carpet company can get market share only by purchasing competitors, not by agrressive marketing.
A.weaken
B.irrevelant
C.irrevelant
D.support
E.support
作者: jetyxo    时间: 2012-6-4 23:28
1 premise: carpet market is tied to the population ages, people only by twice in their age
 conclusion:to extend market share, company can only through purchasing competitors ,not aggressive marketing
  weaken  validity:may be there is other way rather than purchasing competitors to extend market share
 A B C Eirrelevant, D is the answer, company also can get more market share through reducing price to make other competitors out of the market.

2 premise 1: increasing the amount of carbon dioxids absorbed by plant can reduce carbon dioxide
  premise 2: establish  seeweeds farms which will be burn after die
  conclusion: establish  seeweeds farms  will be effect in reducing carbon dioxides
  weaken: weather burning the seeweeds will release more carbon dioxids than they absorbed  ----> B

3\J :video rentals decline more than 10000because  a new rental shop opened this year,which also sell 4000videos to people
  M:there is other reason for the decline
  weaken M, suppot J,the decline is due to the new shop selling 4000 videos--->E, give a reason why 4000 videos result to 10000 decline in video rental

4\ M: new wine will compete with our wine because their bottles look like ours
   D: no , the label is different
   weaken D: may be people will not pay attention to this litter difference---> D

5\ Finding of a magazine subscriber:30%peopel make orders by the ad. are under 35
   Finding of the advertisement company: most of people make orders by the ad. are under 35
  answer: there is a gap between 30% and most
  reason:may be they surveyed different people --->E
作者: bet    时间: 2012-6-5 23:27
六月五日 【1-3】
1.    39”
BG:Most who purchase carpet do so only once or twice, first in their twenties or thirties, and
then perhaps again in their fifties or sixties.
P:The population is aging
C:companies will gain market share only through purchasing competitors, and not through more aggressive marketing.
推测:aging population means more purchase in people’s fifties or sixties
选D ABCE无关

2  B
3  E
4  D
5  E

今天时间实在太紧,马上断网了,仓促看了。。。。
作者: penguinsdz    时间: 2012-6-9 11:12
标题: 6.07+【1-3】
6.07+1-3

1. There is relatively little room for growth in the overall carpet market, which is tied to the size of the population. Most who purchase carpet do so only once or twice, first in their twenties or thirties, and then perhaps again in their fifties or sixties. Thus as the population ages, companies producing carpet will be able to gain market share in the carpet market only through purchasing competitors, and not through more aggressive marketing.

Which one of the following, if true, casts the most doubt on the conclusion above?
(A) Most of the major carpet producers market other floor coverings as well.
(B) Most established carpet producers market several different brand names and varieties, and there is no remaining niche in the market for new brands to fill.
(C) Two of the three mergers in the industry’s last ten years led to a decline in profits and revenues for the newly merged companies.
(D) Price reductions, achieved by cost-cutting in production, by some of the dominant firms in the carpet market are causing other producers to leave the market altogether.
(E) The carpet market is unlike most markets in that consumers are becoming increasingly.
patterns and
Background Information:
There is relatively little room for growth in the overall carpet market, which is tied to the size of the population.
Premise:
Most people who purchase carpet first in their 20 or 30, and then again in their 50 or 60.
Conclusion:
Thus companies producing carpet will be able to gain market share in the carpet market only through purchasing competitors and not through more aggressive marketing.

推测(Prephrase: mentally formulate your answer to the question stem)

The companies producing carpet can gain market share in the carpet market not only through purchasing competitors.

选项分析:D



2. Scientists are discussing ways to remove excess carbon dioxide from the atmosphere by increasing the amount that is absorbed by plant life. One plan to accomplish this is to establish giant floating seaweed farms in the oceans. When the seaweed plants die, they will be disposed of by being burned for fuel.
Which of the following, if true, would indicate the most serious weakness in the plan above?

(A) Some areas of ocean in the Southern Hemisphere do not contain sufficient nutrients to support large seaweed farms.
(B) When a seaweed plant is burned, it releases an amount of carbon dioxide comparable to the amount it has absorbed in its lifetime.
(C) Even if seaweed farms prove effective, some people will be reluctant to switch to this new fuel.
(D) Each year about seven billion tons of carbon dioxide are released into the atmosphere but only about five billion tons are absorbed by plant life.
(E) Seaweed farms would make more money by farming seaweed to sell as nutritional
supplements than
Background Information:
Scientists are discussing ways to remove excess CO2 from the atmosphere by increasing the amount that is absorbed by plant life.
Premise:
One plan is to establish giant floating seaweed farms in the oceans.

Conclusion:
The seaweed farms can absorb the CO2, and they will be disposed of by being burned for fuel after they die.

推测(Prephrase: mentally formulate your answer to the question stem)

Fuel burning produces CO2.

选项分析:B



3. Jennifer: Video rental outlets in Centerville together handled 10,000 fewer video rentals in 1994 than in 1993. The decline in rentals was probably due almost entirely to the February 1994 opening of Videorama, the first and only video rental outlet in the area that, in addition to renting videos, also sold them cheaply.
Brad: There must be another explanation: as you yourself said, the decline was on the order of 10,000 rentals. Yet Videorama sold only 4,000 videos in 1994.
Which of the following, if true, would most seriously weaken the force of the objection that Brad presents to Jennifer's explanation?

(A) In 1994 Videorama rented out more videos than it sold.
(B) In 1994 two new outlets that rent but that do not sell videos opened in Centerville.
(C) Most of the video rental outlets in Centerville rent videos at a discount on certain nights of the week.
(D) People often buy videos of movies that they have previously seen in a theater.
(E) People who own videos frequently loan them to their friends.
Background Information:
Video rental outlets in Centerville handled 10000 fewer video rentals in 1994 than in 1993. Videorama is the first and only video rental outlet in the area that, in addition to renting videos, also sold them cheaply.
Conclusion:
Jennifer: The decline in rentals was due to the February 1994 opening of Videorama.

Brad: There must be another explanation: The decline was on the order of 10000 rentals; however, Videorama sold only 4000 videos in 1994.
选项分析:E



4. Mourdet Winery: Danville Winery's new wine was introduced to compete with our most popular wine, which is sold in a distinctive tall, black bottle. Danville uses a similar bottle. Thus, it is likely that many customers intending to buy our wine will mistakenly buy theirs instead.
Danville Winery: Not so. The two bottles can be readily distinguished: the label on ours, but not on theirs, is gold colored.
Which of the following, if true, most undermines Danville Winery's response?
(A) Gold is the background color on the label of many of the wines produced by Danville Winery.
(B) When the bottles are viewed side by side, Danville Winery's bottle is perceptibly taller than Mourdet Winery's.
(C) Danville Winery, unlike Mourdet Winery, displays its wine's label prominently in advertisements.
(D) It is common for occasional purchasers to buy a bottle of wine on the basis of a general impression of the most obvious feature of the bottle.
(E) Many popular wines are sold in bottles of a standard design.
Background Information:
DW’s new wine was introduced to compete with MW’s most popular wine, which is sold in a distinctive tall, black bottle.
Premise:
DW uses a similar bottle.
Conclusion:

MW: It is likely that many customers intending to buy our wine will mistakenly buy DW’s instead.
DW: The two bottles can be readily distinguished, this is because the label on ours is gold colored.

推测(Prephrase: mentally formulate your answer to the question stem)
Customers usually can’t see the unobvious feature at all.

选项分析:D



5. Finding of a survey of Systems magazine subscribers: Thirty percent of all merchandise orders placed by subscribers in response to advertisements in the magazine last year were placed by subscribers under age thirty-five.

Finding of a survey of advertisers in Systems magazine: Most of the merchandise orders placed in response to advertisements in Systems last year were placed by people under age thirty-five.
For both of the findings to be accurate, which of the following must be true?

(A) More subscribers to Systems who have never ordered merchandise in response to advertisements in the magazine are age thirty-five or over than are under age thirty-five.
(B) Among subscribers to Systems, the proportion who are under age thirty-five was considerably lower last year than it is now.
(C) Most merchandise orders placed in response to advertisements in Systems last year were placed by Systems subscribers over age thirty-five.
(D) Last year, the average dollar amount of merchandise orders placed was less for subscribers under age thirty-five than for those age thirty-five or over.
(E) Last year many people who placed orders for merchandise in response to advertisements in Systems were not subscribers to the magazine.

Background Information:
A survey of subscribers: 30% of all orders placed by subscribers were placed by subscribers under age 35.

A survey of advertisers: most of the orders (假设70%) placed were placed by people under age 35.

推测(Prephrase: mentally formulate your answer to the question stem)
Many orders (
假设40%) placed by people who were not subscribers.
选项分析:E


作者: LuckyYolandaLi    时间: 2012-6-17 22:16
今天我做题感觉不确定的地方有好多。。。

1.
1
)计时:80s
2
)逻辑链:
Premise: Most who purchase carpet do so only once or twice.
Intermediate conclusion: There is relatively little room for growth in the overall carpet market, which is tied to the size of the population.
Conclusion: companies producing carpet will be able to gain market share in the carpet market only through purchasing competitors, and not through more aggressive marketing.
3
)推测:文章的推理我没太理解,所以没有想出合理的推测。。。
4)选项分析:D
A: Irrelevant.
B: 貌似有点strengthen.
C: D: E: 我理解得不好。。。


2.
1)计时:50s
2
)逻辑链:
Premise: Scientists are discussing ways to remove excess carbon dioxide from the atmosphere by increasing the amount that is absorbed by plant life.
Conclusion: One plan to accomplish this is to establish giant floating seaweed farms in the oceans. When the seaweed plants die, they will be disposed of by being burned for fuel.
3
)推测:
Burning the dead seaweed plants will produce carbon dioxide.
4
)选项分析:B
A: Not the most serious.
B: Correct.
C: Not the most serious.
D: Irrelevant.
E: Not weakness.


3.
1)计时:64s
2
)逻辑链:
这种先阐述一个人观点再有另一个人反对的情况,我不知道怎么写逻辑链。。。
3
)推测:
Videorama, selling videos cheaply, attracted many consumers who not only bought videos but also rent videos there.
4
)选项分析:A(正确答案:E
A: Correct.(我原来的理解是Videodrama卖的video便宜,所以卖出去的多,那既然选项说租出的比卖出的还多,所以说明很多人在Videodramavideo,这些人中可能有很多是以前在Centerville租的,所以如此,在Centervillevideo的人就更减少了。但既然我答案选错了,说明可能是我理解错了,但我还是不太明白是哪里错了。。。是没有E好吗?还是根本就不对?
B: Strengthen Jennifer’s explanation.
C: Irrelevant.
D: Irrelevant.
E: Irrelevant. (看完答案后,根据答案自己理解:Videodramavideo便宜,所以很多人不租而直接在这买,这些买了的人又把买的video借给朋友,所以他们的朋友也不用去租video了,所以Centerville租出的video就减少了)


4.
1)计时:41s
2
)逻辑链:
(同上题)
3
)推测:
The customers usually don’t pay attention to the label.
4
)选项分析:D
A: Irrelevant.
B: Strengthen Danville Winery’s response.
C: Strengthen Danville Winery’s response.
D: Correct.
E: Irrelevant.


5.
1)计时:60s
2
)逻辑链:
Premise:Thirty percent of all merchandise orders placed by subscribers in response to advertisements in the magazine last year were placed by subscribers under age thirty-five.
Most of the merchandise orders placed in response to advertisements in Systems last year were placed by people under age thirty-five.
3)推测
这道题我理解不太好,没有推理出来。。。
4)选项分析:E
ABCD不能从文中推出来。
作者: angelafeng    时间: 2012-6-20 15:29
1. 43s

Background informationThere is relatively little room for growth in the overall carpet market, which is tied to the size of the population.

PremiseMost who purchase carpet do so only once or twice, first in their twenties or thirties, and then perhaps again in their fifties or sixties.

ConclusionThus as the population ages, companies producing carpet will be able to gain market share in the carpet market only through purchasing competitors, and not through more aggressive marketing.

Prephraseweakencompanies producing carpet can gain market share in the market through some other ways other than through purchasing competitors.



(A) Most of the major carpet producers market
other floor coverings as well.-----------Other floor coverings have litter relationship with carpet.
(B) Most established carpet producers market
several different brand names and varieties,
and there is no remaining niche in the
market for new brands to fill.-----------that has no bearing on the conclusion of the argument.
(C) Two of the three mergers in the industry’s last
ten years led to a decline in profits and
revenues for the newly merged companies.------------ the above argument has no idea about the mergers.
(D) Price reductions, achieved by cost-cutting in
production, by some of the dominant firms
in the carpet market are causing other
producers to leave the market altogether.--------------suggest a way for companies to gain market share without purchasing competitors, thereby attacking the conditional statement given in the stimulus.

(E) The carpet market is unlike most markets in
that consumers are becoming increasingly
resistant to new patterns and styles.-------------CORRECT-----------strengthen the argument



2.28s

Premise: One plan to accomplish this is to establish giant floating seaweed farms in the oceans.  When the seaweed plants die, they will be disposed of by being burned for fuel.

Conclusion : Scientists think that this is a way to remove excess carbon dioxide from the atmosphere by increasing the amount that is absorbed by plant life.

Prephrase: (weakness) burn the dead seaweed plants for fuel will produce more co2 into the atmosphere.

Answer: B



3.44S

Premise: Video rental outlets in Centerville together handled 10,000 fewer video rentals in 1994 than in 1993.  

Jennifer’s explanation: Due to the February 1994 opening of Videorama, the first and only video rental outlet in the area that, in addition to renting videos, also sold them cheaply.

Brad’s refute:  Videorama sold only 4,000 videos in 1994.

Prephrase: (weaken Brad) people always share the videos with their friends.

Answer: E



4.34s

Premise:

MW---Danville Winery's new wine was introduced to compete with MW’s most popular wine, which is sold in a distinctive tall, black bottle.  Danville uses a similar bottle.  It is likely that many customers intending to buy MW’s wine will mistakenly buy DW’s instead.

DW--Objection  Not so.  The two bottles can be readily distinguished:  the label on ours, but not on theirs, is gold colored.

Prephrase:( weaken DW) many buyers don’t have a look at the label of the wine bottles when they buy wine.

Answer: D



5.37S

Premise 1:  Thirty percent of all merchandise orders placed by subscribers in response to advertisements in the magazine last year were placed by subscribers under age thirty-five.

Premise 2:  Most of the merchandise orders placed in response to advertisements in Systems last year were placed by people under age thirty-five.

Prephrase: (conclusion) Most merchandise orders placed by subscribers in response to ads in the magazine last year were placed by subscribers under age 35.

Answer:C E

(C) Most merchandise orders placed in response to advertisements in Systems last year were placed by Systems subscribers over age thirty-five.------------under age 35

作者: 呵呵牙    时间: 2012-6-22 06:31
昨天有事耽误了~今天补上~我一定要坚持!!!

1. 33s
Background Information: There is relatively little room for growth in the overall carpet market, which is tied to the size of the population.
Premise: People buy carpet do so only once or twice, first in their twenties or thirties, then again in fifties or sixties.
Conclusion: Company producing carpet will be able to gain market share only through buying competitors, and not through more aggressive marking
Prephrase: Even though people seldom buy carpet, customer’s also important key to make company gain market share; buying competitors’ carpet is not the only way to gain market share.
选D
A (major carpet producers, coverings)——irrelevant
B (several different brand names and varieties)——support conclusion
C (company mergers)——irrelevant
E (carpet market different)——support conclusion

2. 32s
Background Information: Scientists discuss ways to reduce carbon dioxide from atmosphere.
Premise: One plan to finish this is to establish seaweed farms in the oceans.
Conclusion: Carbon dioxide will be disposed after the seaweed plants die.
Prephrase: Seaweed plants can die and grow again; to burn the death of seaweed plants also can produce carbon dioxide.
选B

3. 45s     !!!求解释,我不明白是weaken谁的观点???(最近脑子不太好使……)
Premise: <J>Rental decline because the February 1994 opening of Videorama and cheap price to rental.
             <B>The decline was on the order, but Videorama sold only 4,000 video in 1994.
Conclusion: Video rental outlets in Centerville handled decline in 1994 than in 1993
Prephrase: Videorama sold less but they make more money from selling videos.
选E

4. 38s
<M>remise: Our popular wine is tall, black bottle. D use a similar bottle.
Conclusion: Customers want to buy ours, but make mistake to buy D’s wine.
<D>remise: Our bottle has label that is gold colored.
Conclusion: Customers can be distinguished
Prephrase: Label is confusing to people.
选D

5. 40s
<subscribers>remise: Under 35 age, 30% of orders placed by subscribers in response to advertisements.
<advertisers> Premise: Under 35 age, most of orders placed in response to advertisements in Systems.
Prephrase: Under 35 age, subscribers and advertisers are seldom in response in System magazine.
选E
作者: jiajiajudy    时间: 2012-6-23 05:33
1.Only one or twice in their life?Little growth room for carpet bcuz of the size of population?gain market share instead of through purchasing competitors

(D) Price reductions, achieved by cost-cutting in
production, by some of the dominant firms
in the carpet market are causing other
producers to leave the market altogether.

2.Floating seaweed forms(which can be burned for fuel after die)?increase amount of plant to remove excess co2

(B) When a seaweed plant is burned, it releases an amount of carbon dioxide comparable to the amount it has absorbed in its lifetime.

3.J:Vieorama open, which is the only outlet in that area, also sold cheap?Centerville 1994<1993
B:disagree. Rental decline, V sold 4000 in 1994

(B) In 1994 two new outlets that rent but that do not sell videos opened in Centerville.
4.MW: Customer mistakenly buy DW because of the similar bottle
DW: not agree  lebel I gold colour
(D) It is common for occasional purchasers to buy a bottle of wine on the basis of a general impression of the most obvious feature of the bottle.

5.Finding of a survey of Systems magazine subscribers: Thirty percent of all merchandise orders placed by subscribers in response to advertisements in the magazine last year were placed by subscribers under age thirty-five.

Finding of a survey of advertisers in Systems magazine: Most of the merchandise orders placed in response to advertisements in Systems last year were placed by people under age thirty-five.

For both of the findings to be accurate, which of the following must be true?


(D) Last year, the average dollar amount of merchandise orders placed was less for subscribers under age thirty-five than for those age thirty-five or over.
作者: cleotina    时间: 2012-6-26 17:09
6-26
50''
Q:weaken
B: people purcuse carpet certrilizing on two age area. and the person

prechase the carpet just one or twice during their life time. once in

twenties or in thirties, the other in 50 or 60.
P:as population ages, the way they want to enlarge their market is

purchasing competitors, but not through more aggressive marketing.
guess:
1 as the population ages, more customer who never bought a carpet before ,

they choose to buy one. So, aggressive marting will enlure more new aged

customers
2 not all the young person will buy during theri 20, company can enlarge

their customers through advertisment
(1,这种猜测是错的,因为和所给的背景相反。 不能假定背景的错误,一方面在阅读的时候要抓住信息  2,要抓住结论的侧重点,此处的结论是为了提高市场占有率,在此基础上探讨方法 3, 如果其他的方法有效就消除了文中提出的方法的有效性)

OA: C there were several companies that mergered some competitors, however,

they lost

A:Irrelevant, it is not depend on other produces
B:there is no nitch market   Irrelevant
D:strong
E:about the perference of the carpet customes. sound ok, bur still

irrelevant.


26''
Q:weaken
B:scientists try to find a way to eliminate the excess CO2 in the

atmosphere
P: Since plant in the ocean will inspire CO2 of the atmosphere
C:breed more plant, after they die, buring for fuel as the way to dispose
Guess:
1,more carbon will be discharged
2, ocean is not the area with high level of co2
OA:B
A: irrelevant: without enough noutrious to feed the plant
C: weaken, but no the main point of the argument
D: comparision, irrelevant
E: irrelevant  about the economic way to dispose the plant

49''
weaken P2
B:few than 10000 vedos rental in one place in 1994 than in 1993, that because a new outlet open, in addition to renting, it sold them cheaply, which is the only one do
P: other reason may also lead to the deciline, since the new outlet sale

4000, which is less than 10000 , in 1994
Guess:
except sell the new store also rent, so, others may choose to rent one

rather than buy a new one
OA: a they rent more than they sell
B: strong
C: irrelevant
D: strong
E: strong
BDC , give a new explains to the decline, which is support the argument of

2 person
(做错了,把10000个借出量的降低理解为了销售量的降低,stupid)


44''
weaken 2
M: D use the same distinctive bottle similer to a kind of high saleble wine

of M, to cheat customers to buy their wine
D: the too kind of bottle can easily distincted since D has a golden logo

in the bottle, while the bottle of M without
Guess:
1 it is a customs that when peopkle buy a wine they choose it depen on the

bottle
2 the golden logo in the bottle is in the back of the bottle, or D hinden

it when dispose in the shelf
OA
A : Irrelevant
B strong
C: irrelevant
E: strong


50''
whiche of the following must be true
P1: 30% subscribers who order in the reponse to the newspaper were under 35
P2: most of the subscribers who response to advertisements in the systems were people under 35
Guess:
1, most of the people under 35 who odering are reponse to others kinds of advertises
OA:E
作者: 非同反想    时间: 2012-7-11 16:27
问一下 精炼题中c选项的具体意思是什么呀。。


Two of the three mergers in the industry’s last
ten years led to a decline in profits and
revenues for the newly merged companies.
作者: Donts    时间: 2012-7-14 10:10
1.计时1:00
premise a person may only purchase carpet twice during its lifetime--> the market is limited to the population;the pop. ages
conclusion producer can reap a profit by purchasing competitors rather than by marketing
推测acquisition&merge并没有带来更好的结果
(A) Most of the major carpet producers market
other floor coverings as well.
与purchasing competitor无关
(B) Most established carpet producers market
several different brand names and varieties,
and there is no remaining niche in the
market for new brands to fill.
new brands无关
(C) Two of the three mergers in the industry’s last
ten years led to a decline in profits and
revenues for the newly merged companies.那还有三分之一有机会牟利。
(D) Price reductions, achieved by cost-cutting in
production, by some of the dominant firms
in the carpet market are causing other
producers to leave the market altogether.与purchasing无关
(E) The carpet market is unlike most markets in
that consumers are becoming increasingly
resistant to new patterns and styles.

选择E
2.计时30''
backgroundscientists are looking for a method to decrease co2 in the atmosphere by increase the amount absorbed by plant life.
premise某种植物可以吸收co2
conclusion种植这种植物去吸收二氧化碳,死了以后还可以burned for fuel
推测看到burn就条件反射想到二氧化碳,所以burn也会带来co2的话就可以削弱
选择B
3.
计时43''
J:Video rental in C declined in 1994 because a newly opened video rental not only rent but also sell in a cheap price.
opponent: the declined number is in the order of 10000, however, the new shop only sold 4000
推测:卖出的4000有别的效应,比如卖出去的人邀请朋友到家里,所以都看过
选择E
4.
计时45‘’
MWW use a similar bottle with our most popular wine bottle, some consumers may mistakenly buy their wine
DW: its wrong, the two brand are in gold which is clear enough for consumers to separate two different brands.
推测
选择D
5.
计时42''
A30% purchase orders by subscribers in response to ad. were replaced by subscribers under 35
B most purchase orders in response to ad. last year were replaced by consumers under 35
推测答案和consumer以及subscriber有关。不是所有的consumer in response to ad.都是consumer
选择E
订正1.D
总结To attack a conditional statement you must show that the necessary condition is not actually necessary for the sufficient condition to occur.
作者: silviahelena    时间: 2012-7-31 22:14
打卡,等会回来发逻辑链

精练题果然错了。。。我选了E

先分析后4题的逻辑链

2. Premise: Increase amount of plant can reduce carbon dioxide
   Conclution: By establishing giant floating, and burning died seaweed as fuel, the amount of carbon dioxide can be reduced
   Weaken: If burning the seaweed will generate more carbon.
   Answer is B

3. Premise: together handled 10k fewer rental in 1994 than 1993. the videorama sold 4K.
   Conclution: as 4k<10k, the decrease is not due to videorama selling the videos
   Weaken: if the 4k sold can be lended to others, the demand will also decrease
   Answer is E
4. Premise: the competitor uses a similar black bottle. D's bottle use a golden label
   Conclution: customer can differentiate by the label
   Weaken: if the label doesn't help to distiguish
   Answer is D
5. Premise:30% of subscribers under 35 placed the order; most ppl placed order are under 35
   Explanation: total number of subscribers > the number of advertisement audience..
   Answer is E
脑子已经不转了,请允许我明天再重新想第一题。。。。。。。不好意思
作者: TICKCOCK    时间: 2012-8-2 20:06
1、    little room for growth in the overall carpet market.
Premise: most customers buy carpet only once or twice
Conclusion: purchasing competitors rather than aggressive marketing is the better solution to gain market share.
Answer: C(D)

2. background: in order to remove excess carbon dioxide, one solution is to increase the amount that is absorbed by plant life.
Premise: one solution is the plan establish giant floating seaweed farms in the ocean.
Conclusion: seaweed die, they will be disposed of by being burned for fuel.
Answer :B

3.Jennifer argues that the decrease of rentals in C is entirely blame to opening of videorama which not only rents video but also sells videos at lower price.

Brad refutes that by saying must be other explanation. Because, the amount V sold in 1994 only 4000, can’t compare to 10000 decline of rental.
Answer: E

4. Mourdet Winery: consumers would mistakenly take D’s wine as M’s wine since both of them are sold in a distinctive tall, black bottle.
Danille Winery objects: label are not the same.

Answer

5.Finding of a survey of systems magazine subscribers: 30% of all merchandise orders were placed by subscribers under age 35
Finding of a survey of advertisers in systems magazine: most of merchandise orders placed in systems were placed by people under 35.
Answer: E
作者: wanggang0411    时间: 2012-8-8 16:33
1    36
Conclusion: The only way to gain carpet market share is to purchase other carpet company, other aggressive marketing doesn’t work.
Inference:  Given any example to testify that other aggressive marketing is work will weaken the conclusion.
2    38
Premise: Scientists plan to grow plant to absorb D in order to reduce the dioxide in atmosphere.
Conclusion: Scientists will grow S farms in oceans, and this S farms can be disposed by fuel.
Inference: The dioxide will be generated during the combustion of S farms, and then emit to atmosphere.
3    49
J:    Video rentals are 10000 fewer in 1993 than that in 1994.
This decline is caused the opening of VC.
B:    I disagree since VC only sold 4000 video last year.
Inference: another reason explained this 4000 video sold caused the 10000 rentals decline directly or indirectly.
4    `42
MW: The bottle of wine sod by DW is similar with that sold by MW, so customer of MW will buy DW’s wine mistakenly.
DW: No. The color of label is different.
Inference: Any factors indicating that the only different in color will not avoid the mistaken purchase will undermine DW’s response.
5    35
Finding A: 30% merchandise orders were placed by subscriber under age thirty-five.
Finding B: Most merchandise orders were placed by people under age thirty-five.
Inference:
作者: FannyL    时间: 2012-8-16 14:08
1.38' Weaken
background:the growth in the overall carpet market is little,as the size of population
premise:companies will gain market share through purchasing competitors
conclusion:to get the market share more.
only by purchasing competitor,but do not improve self-conpetitioni,and do not consider improve the purchase
A:irrevelent.
B:即使有different brand names ,也无法解释conclusion
C:不充分
D:rigt
E:无法解释。
2.30' Weaken
background:scientist try to remove excess carbon dioxide by increasing the plant to absorb it
premise:establish giant floating seaweed farms in the oceans,and it can become fuel though burning

establish giant floating in the ocean may cause the pollution of ocean,like O2
在AB中徘徊,但是觉得要契合题干 所以选B
A:从ocean角度说,但是并没有解决carbon dioxide的排放问题
B:指出了文章的漏洞
C:irrevelent
D:从金钱角度不能说明排放量的问题
E:同D
3.40'
background:rental outlet in 1994 is 10,000fewer video than in 1993
premise:the opening of Videorama
选A 这道题逻辑链没有怎么找出来
4.30’
premiseancille's wine' bottle is similar to our popular wine's, competition
conclusin:customs may be mistaken
选D
5.第五题没有理解到。。。。。先放这。。再分析。。
作者: chengzaaaa    时间: 2012-8-20 21:47
3.
1)38”weaken
2)BI: no room for carpet market growth. Tied to the size of population.
P: people buy carpet two times in their life.
C: gain market shares only through purchasing competitors, not through aggressive marketing.
3) population structure ensures that there are people who want to buy carpet all the time…….
4)D
5)A. Range. Other floor coverings are irrelevant.
B. new brand is irrelevant.
C. not weakening…….
D. Correct. Price reductions, more market share, not purchasing competitors.
E. not weakening………..
T_____T

2.
1)28”weaken
2)BI: try to remove CO2, planting more plants, absorbing the CO2
P: plant seaweed farm in the oceans. They die, burn them for fuel.
C: seaweed farm can absorb the excess CO2.
3) burning the farm produce more CO2 than the farm absorbs
B

3.
1)34”weaken
2)BI: 1994=1993-10,000. 1994,open new place, not only rent, but also sell.
P: decline:10,000. Sold:4,000
C: must be another explanation.
3) people who buy the videos also lend them to frds. So the total rent number declined,
E

4.
1)23”weaken
2)BI: D has new wine out. Looks like M’s bottle. M says people intend to buy theirs and mistakenly buy D’s.
P: D’s labels are gold colored.
C says that two bottles can be distinguished. Customers are not mistakenly buying.
3) people cant really see the gold part. They generally see the shape of the bottle.
D

5.
1)31”
2)
3)most of the orders are not placed by subscribers.
E


谢谢lz!!!
作者: Maykey    时间: 2012-8-25 17:17
1-3
3.45S
BG: there is little room for carpet market as it is tied to the overall polulation.
Premise: People will buy carpet on their twenties or thirties and fifties or sixtie.
Conclusion : carpet company can gain it market shares only though purchasing competitors not though more aggressive market
推测:may be aggressive market can bring great share to the companies,may be it is just a tome problem.
A,无关
B.无关
C,support
D,correct, because other producers leave the market, dominant firm will gain larger market share.
E .无关
2.    BG:To increase excess CD from atmosphere,scientists increase the amount that absorbed by plant.
3.    remise: the plan is to establish seeweed farm
4.    Conclusion: when seaweed die ,they will be burned for fuel
The burning of seaweed will produce a large amount of CD
A 无关
B,correct
C.无关
D,not strong enough to weaken
E无关
作者: 我心匪席    时间: 2012-8-29 16:04
1)    Time: 34”
2)    Situation:
Background: The carpet marketing is almost full. Customers usually purchase twice, 20 or 30, then 50 or 60.
Premise: Carpet Company gains its market share.
Conclusion: Merging competitor companies.
3)    rephrase: customers either buy from this carpet marketing or buy from competitors.
4)    A----irrelevant
B----support
C----support (C-----P)
D----right
E----support
========================================
1)    Time: 24”
2)    Situation:
Premise: Remove excess carbon dioxide from the atmosphere.
Conclusion: Establish seaweed in the ocean, burn it after die.
3)    rephrase: Burning plants will cause more carbon dioxide.
4)    A----irrelevant
B----right
C----irrelevant
D----irrelevant
E----irrelevant
=========================================
1)    Time: 37”
2)    Situation: Video rentals decreased in C from 1994 to 1993, most due to the opening of V, who rent and sell videos.
Premise: V sold 4000 videos in 1994, and rentals only declined 10000.
Conclusion: Do not blame V
3)    rephrase: Customers buy videos instead of renting them.
4)    A----support
B----irrelevant
C----irrelevant
D----support
E----right
========================================
1)    Time: 30”
2)    Situation:
Premise: Label is gold colored
Conclusion: Two brands can be distinguished only by label color.
3)    rephrase:
4)    A----irrelevant
B----irrelevant
C----irrelevant
D----right
E----irrelevant
========================================
1)    Time: 30”
2)    A----irrelevant
B----irrelevant
C----irrelevant
D----irrelevant
E----right
作者: huang2610    时间: 2012-9-3 23:36
P: market for carpet is very small and relating to the population. Cos most of the people only purchase twice during the life time

C: Companies only can gain the market growth from the the competitor instead of from marketing

Weaken

方向:新型营销策略可以改变人们的购买习惯
P:S suggest to grow more plants to absorb the CD
C: one method will be grow the some kind of seaweed to absorb the CD and when seaweed die, use them as a fuel.
Weaken: Burn seaweed will create more CD then they absorb.

P: J: Video rental decline 10000 1994 compare with 1993.
C: J: the decline is due to the videorama sales the video cheaply

Counter P: B: the Videorama only sold 4000 CD but the rental decline was 10000-Much bigger
Counter C: B: Videorama is not the reason for the decline

Weaken: Rental has time limit, after watch customer have to return the CD on time, But purchase CD will allow them to lend it to their friends

P: MW: the DW’s wine  using same design one kind of our wine’s bottle design
C:MW: this will mislead some of our customer who intent to buy our wine end up pick up their product

DW: P; our label is golden colour but yours not
DW:C:it won’t be a problem for customer to distinguish the two brands
Weaken: Most of the customer doesn’t realize that the MW having a similar wine, when they buy, the never read the label carefully
作者: huang2610    时间: 2012-9-3 23:46
第五题云里雾里,跟同学们相比弱爆了  加油
作者: ElenW    时间: 2012-10-2 09:59
我也来跟一个
精练3  49.3S
pre:first in their twenties or thirties, and then perhaps again in their fifties or sixties.
conclusion:Thus as the population ages, companies producing carpet
will be able to gain market share in the carpet market only through purchasing competitors, and not  through more aggressive marketing.
预测:weaken the conclusion . The goal  will not be achieved
Ans
2  22'3
pre:plan:remove the carbon dioixde from the atmosphere by incresing the amount that is absorbed.
conclusion:when plants die ,they will be disposed of by being burned for fuel.
预测:weaken this plan 可以移除carbon dioxide 但是有的地方没有具备种植这个Plants的条件 或者这样的做法根本不会有成效.
Ans:B
3  49'9
Bg:Video rental outlets in Centerville together handled 10,000 fewer video rentals in 1994 than in 1993.
explain1:It was  probably due  alomst entirely to the February 1994 opening of Video drama, the first and only video rental outlet in the area that, in addition to renting videos, also sold them cheaply.
explain2:the decline was on the order of 10,000 rentals.  Yet Videorama sold only 4,000 videos in 1994.
预测:不能去否定explain1 对于explain2中 没有考虑到total amount 的问题
Ans:A
4  40's
Bganville Winery's new wine was introduced to compete with our most popular wine
prean. use a similar bottle.
pre:the way to distinguish them:the label on ours, but not on theirs, is gold colored.
conclusion:it is likely that many customers intending to buy our wine will mistakenly buy theirs instead.
预测:她只说了在他们桌上的是gold色的 ,但是在别人桌上的任然是很相似的 依然会被认错,更和空大家的bottle 是很相似的.
Ans
5 32'7s
pre1:30% orders placed by subscribers by subscribers age<35
pre2:Most   by people <35
conclusion:bothe of them are true.
预测:
Ans:E
作者: icefiremin    时间: 2012-10-4 11:12
3. There is relatively little room for growth in the
overall carpet market, which is tied to the size of the
population. Most who purchase carpet do so only
once or twice, first in their twenties or thirties, and
then perhaps again in their fifties or sixties. Thus as
the population ages, companies producing carpet
will be able to gain market share in the carpet market
only through purchasing competitors, and not
through more aggressive marketing.
Which one of the following, if true, casts the most
doubt on the conclusion above?
--BIBLE WEAKEN NO.3


40’+1’56
P: people only buycarpet at 20/30 and 50.
C: little room forgrowth of carpet market
C: company can onlygain edge through purchasing competitors rather than marketing.
Pre: marketing changepeople’s mind when buy at 20/30 and 50.

(A) Most of the major carpet producers market
other floor coverings as well. ---------Irrelevant.
(B) Most established carpet producers market
several different brand names and varieties,
and there is no remaining niche in the
market for new brands to fill. ----------Irrelevant.
(C) Two of the three mergers in the industry’s last
ten years led to a decline in profits and
revenues for the newly merged companies. --------------It’s about market share,not profit/revenue.
(D) Price reductions,achieved by cost-cutting in
production, by some of the dominant firms
in the carpet market are causing other
producers to leave the market altogether.
----------------Other causescan have the same effect.
(E) The carpet market is unlike most markets in
that consumers are becoming increasingly
resistant to new patterns and styles. -------------Not about new/old patterns,about existing competition.

【逻辑链】
2.
Scientists are discussing ways to remove excess carbon dioxide from theatmosphere by increasing the amount that is absorbed by plant life.  Oneplan to accomplish this is to establish giant floating seaweed farms in theoceans.  When the seaweed plants die, they will be disposed of by beingburned for fuel.

Which of the following, if true, would indicate the mostserious weakness in the plan above?
-- PREP07 CR1 NO.9

P: produce seaweed

P: seaweed is burned for fuel when die.
C: reduce the amt of CO2 by increasing amt absorbed by plant life.
Pre: burning has negative effect
27’+31’
(A) Some areas of ocean in the Southern Hemisphere do notcontain sufficient nutrients to support large seaweed farms.----Irrelevant
(B) When a seaweed plantis burned, it releases an amount of carbon dioxide comparable to the amount ithas absorbed in its lifetime.
(C) Even if seaweed farms prove effective, some people will be reluctant toswitch to this new fuel.-----“some people” not effective weakening.
(D) Each year about seven billion tons of carbon dioxide are released into theatmosphere but only about five billion tons are absorbed by plant life.----Justprevious fact.
(E) Seaweed farms would make more money by farming seaweed to sell asnutritional supplements than by farming seaweed to sell as fuel.----Irrelevant.


3.
Jennifer:  Video rental outlets in Centerville together handled 10,000fewer video rentals in 1994 than in 1993.  The decline in rentals wasprobably due almost entirely to the February 1994 opening of Videorama, thefirst and only video rental outlet in the area that, in addition to rentingvideos, also sold them cheaply.


Brad:  There must be another explanation:  as you yourself said, thedecline was on the order of 10,000 rentals.  Yet Videorama sold only 4,000videos in 1994.


Which of the following, if true, would most seriously weaken the force of theobjection that Brad presents to Jennifer's explanation?
-- PREP07 CR1 NO.10

P: C has declined rental videos from 1993 to 1994of 10,000. The new opened V has 4,000 sold vedios.
C: Not because of opening of V.
Pre: People switch from rent to buy and lend toother people.
43’+ 27’

(A) In 1994 Videorama rented out more videos than it sold.
(B) In 1994 two new outlets that rent but that do not sell videos opened inCenterville.
(C) Most of the video rental outlets in Centerville rent videos at a discounton certain nights of the week.
(D) People often buy videos of movies that they have previously seen in atheater.
(E) People who own videosfrequently loan them to their friends.


4.
Mourdet Winery:  Danville Winery's new wine was introduced to compete withour most popular wine, which is sold in a distinctive tall, black bottle. Danville uses a similar bottle.  Thus, it is likely that manycustomers intending to buy our wine will mistakenly buy theirs instead.


Danville Winery:  Not so.  The two bottles can be readilydistinguished:  the label on ours, but not on theirs, is gold colored.


Which of the following, if true, most undermines Danville Winery's response?
-- PREP07 CR1 NO.11

P: DW new wine was introduced, the label isgolden, not like the other brand.
C: people not buy DW’s wine because of mistakingit as the other brand.
Pre: Colors are close.
33’+1”

(A) Gold is the background color on the label of many ofthe wines produced by Danville Winery. -----Irrelevant.
(B) When the bottles are viewed side by side, Danville Winery's bottle isperceptibly taller than Mourdet Winery's. -------Irrelevant.
(C) Danville Winery, unlike Mourdet Winery, displays its wine's labelprominently in advertisements.---------people are impressed by the label, can’tmistake it as the other.
(D) It is common foroccasional purchasers to buy a bottle of wine on the basis of a generalimpression of the most obvious feature of the bottle.---------Color isthe most obvious feature.
(E) Many popular wines are sold in bottles of a standard design.----------Irrelevant.


5.
Finding of a survey of Systems magazine subscribers:  Thirty percent ofall merchandise orders placed by subscribers in response to advertisements in themagazine last year were placed by subscribers under age thirty-five.


Finding of a survey of advertisers in Systems magazine:  Most of themerchandise orders placed in response to advertisements in Systems last yearwere placed by peopleunder age thirty-five.


For both of the findings to be accurate, which of the following must be true?
-- PREP07 CR1 NO.12


F:30% of people subscribe commodities orders bysubscribers responding to ad. are <35.
F: most of commodities ordered by people respondingto ad. are <35.
Pre: 30% of people order commodities <35.
44’+1”48’
Go over percentage part in CR Bible.
A) More subscribers to Systems who have never orderedmerchandise in response to advertisements in the magazine are age thirty-fiveor over than are under age thirty-five.
(B) Among subscribers to Systems, theproportion who are under age thirty-five was considerably lower last year thanit is now.
(C) Most merchandise orders placed in responseto advertisements in Systems last year were placed by Systems subscribers overage thirty-five.
(D) Last year, the average dollar amount ofmerchandise orders placed was less for subscribers under age thirty-five thanfor those age thirty-five or over.
(E) Lastyear many people who placed orders for merchandise in response toadvertisements in Systems were not subscribers to the magazine.------link subscriber with people.

作者: srafcatt    时间: 2012-10-4 12:43
标题: [1-3]
精炼:35s weaken
BG:carpet marketing is limited.It is relative to the population.
Premise:people only use carpet when they are fifty to sixty years old or at certain young age.
Conclusion: carpet corporations must purchase their competitors to overcome the difficulty.
If it is not necessary to purchase competitors then the argument will be attacked.
choice
A irrelevant information,the focus is carpet market,not another floor covering.
B strenghten the plan
C strenghten the plan,cause the profits and revenue have already shrinked.
D weaken the plan,corporations don't have to merge eachother,the best corporation will stay and control the market by forcing other corporations to leave through price reduction.
E strenghten the plan
本题的启示:
1.无关的选项通常都是谈到一个相关的话题,但并非是结论中的中心。抓住结论的关键词(topic)无关的选项就会被排除,本题的无关选项是A。
2.shell game(小把戏,抑或叫诡辩) 的识别:通常这种把戏会针对结论中的细节下手,本题C项是将结论中的topic:gain market share 换成了 increase profits and revenues。另一种可能是这种诡辩会对推理的方向下手,本题C项就是将推理方向调换,如果并购,那么利润会上升。而结论中需要反驳的推理是如何证明gain market share--->purchase competitors 是错的。这在逻辑上将其实就是逆命题和命题的真假并不总是同步的。
原题如下:供参考之用
3. There is relatively little room for growth in the
overall carpet market, which is tied to the size of the
population. Most who purchase carpet do so only
once or twice, first in their twenties or thirties, and
then perhaps again in their fifties or sixties. Thus as
the population ages, companies producing carpet
will be able to gain market share in the carpet market
only through purchasing competitors, and not
through more aggressive marketing.
Which one of the following, if true, casts the most
doubt on the conclusion above?
--BIBLE WEAKEN NO.3
(A) Most of the major carpet producers market
other floor coverings as well.
(B) Most established carpet producers market
several different brand names and varieties,
and there is no remaining niche in the
market for new brands to fill.
(C) Two of the three mergers in the industry’s last
ten years led to a decline in profits and
revenues for the newly merged companies.
(D) Price reductions, achieved by cost-cutting in
production, by some of the dominant firms
in the carpet market are causing other
producers to leave the market altogether.
(E) The carpet market is unlike most markets in
that consumers are becoming increasingly
resistant to new patterns and styles.

逻辑链
2. 37s weaken
BG:scientists are willing to make the plant absorb more carbon in order to reduce the carbon element quantity floating in the air.
Premise: establish seaweed frams in the ocean,and burn it as a type of fuel
Conclusion: the plan will succeed:excess carbon dioxide will be removed from atmosphere.
If the seaweed is burned and producing carbon dioxide again,the plan fails.
choice:B
A irrelevant,the plan can be executed if there exists eough valid ocean areas for farms.
B correct
C irrelevant,because we can find another way to use these seaweed.
D strenghten the plan.because the situation is tough,we must deal with the problems by farming seaweed.
E irrelevant information
3.65s two-speaker stimulus weaken
BG:J said in 1994,the video rentals decline.
Premise:because the outlet sell videos cheaply,
Consquence:the rental declines.
another explanation:the quantity of video being sold is much less than the rental.
weaken Brass is strenghten Jennifer.
choice:E
A repeat the materials
B irrelevant information
C irrelevant information,so they buy videos in certain nights,and the total year's selling records are still not changed.
D irrelevant information,it has nothing to do with the way or the stimulus to buy videos.The key problem is why selling videos lead to the rental decline.
E correct.because selling videos will broadly decline the the number of potential customers who want to see videos.
本题启示:错选无关信息的选项常常是因为造成了错误的遐想。
克服错误的遐想的方式是:明确谈论对象,将对象间的逻辑简化。本题逻辑:反驳光碟卖出数量的上升无法填补出租数量下降。因此,辩论关键是证明光碟卖出数量是可以弥补出租数量下降的。

4.36s two-speaker stimulus weaken
MW;premise;the bottle of the two different wines are similar
  conclusionur sales record declined.
DW:premise:the bottles can be distinguished easily,cause the labels are different.

what if the customer seldom look at the label when they choose the wine?
choice
A irrelevant information
B wrong statement,the materials have already told the bottle are simmilar.
C irrelevant information,the situation is taliking about the bottle,not other ways to sell wines.
D correct
E irrelevant information

5.43s fact test
fact1:30% customers order according to adds are under age 35.
fact2:most of all the customers ordering according to adds are under age 35.
maybe other customers' ages are seperated well in vast range.
choice:E
A irrelevant information,it has nothing to do with the subscribers who did't order merchandise after seeing adds.
B This maybe true but not necessarily.
C if this is true,the facts listed upwards are wrong.
D irrelevant statement
E correct,if this is true than it is reasonable that both fact1 and fact2 are true.
作者: mirare    时间: 2012-10-20 08:39
Background
Overall carpet market is tied to the size of population.
Premise
Most purchase carpet once or twice, in their 20s,30s and 50s, 60s.
Conclusion
Purchasing competitors, not through marketing
Prophase
Either-or choice- adjusting
D
作者: xiongyz    时间: 2012-11-9 00:12
1.    33s
P: There is relatively little room for growth in the overall carpet market, which is tied to the size of the population. Most who purchase carpet do so only once or twice, first in their twenties or thirties, and then perhaps again in their fifties or sixties.
C: Thus as the population ages, companies producing carpet will be able to gain market share in the carpet market only through purchasing competitors, and not through more aggressive marketing.
Weaken: 在不同时段都有很多二十或是五十左右想买carpet的人
(A) Most of the major carpet producers market other floor coverings as well. ---irrelevant
(B) Most established carpet producers market several different brand names and varieties, and there is no remaining niche in the market for new brands to fill. ---support
(C) Two of the three mergers in the industry’s last ten years led to a decline in profits and revenues for the newly merged companies. ---support。。。答案说C是shell answer, 为什么呢…
(D) Price reductions, achieved by cost-cutting in production, by some of the dominant firms in the carpet market are causing other producers to leave the market altogether. ---correct,给出了另一个不应该靠价格取胜的原因…
(E) The carpet market is unlike most markets in that consumers are becoming increasingly resistant to new patterns and styles. ---support,对新款式抵抗,所以不应该营销

2.    30s
P: One plan to accomplish this is to establish giant floating seaweed farms in the oceans.  When the seaweed plants die, they will be disposed of by being burned for fuel.
C: Scientists are discussing ways to remove excess carbon dioxide from the atmosphere by increasing the amount that is absorbed by plant life.
Weaken: burn产生的CO2比植物吸收的更多
(A) Some areas of ocean in the Southern Hemisphere do not contain sufficient nutrients to support large seaweed farms. ---irrelevant
(B) When a seaweed plant is burned, it releases an amount of carbon dioxide comparable to the amount it has absorbed in its lifetime. ---correct, weaken, 这样种植物的效果就没有了
(C) Even if seaweed farms prove effective, some people will be reluctant to switch to this new fuel. ---somehow support
(D) Each year about seven billion tons of carbon dioxide are released into the atmosphere but only about five billion tons are absorbed by plant life. ---不能weaken通过植物减少CO2的结论,反而支持,因为确实减少了CO2的量
(E) Seaweed farms would make more money by farming seaweed to sell as nutritional supplements than by farming seaweed to sell as fuel. ---无关

3.    41s
Jennifer:  : Video rental outlets in Centerville together handled 10,000 fewer video rentals in 1994 than in 1993. the first and only video rental outlet in the area that, in addition to renting videos, also sold them cheaply.
C: The decline in rentals was probably due almost entirely to the February 1994 opening of Videorama
Brad:  : as you yourself said, the decline was on the order of 10,000 rentals.  Yet Videorama sold only 4,000 videos in 1994.
C: There must be another explanation
Weaken B: 另外6000是这4000个买的时候同时租出去的~
(A) In 1994 Videorama rented out more videos than it sold. ---support, correct,解释了另外6000也是V造成的
(B) In 1994 two new outlets that rent but that do not sell videos opened in Centerville. ---无关
(C) Most of the video rental outlets in Centerville rent videos at a discount on certain nights of the week. ---与数量无关
(D) People often buy videos of movies that they have previously seen in a theater. ---out of scope
(E) People who own videos frequently loan them to their friends. ---support B
正确答案:E  因为A只说租出的比卖出的多,不能说明总共数量超过10000,所以不能支持entirely, 而且不能直接削弱Brad对于sold数量问题的怀疑…而E,买到之后借给朋友能够解释少了租出的部分,所以削弱Brad说的因为只卖出4000就不能归因于opening of Videorama。。。。

4.    26s
Mourdet Winery:  Danville Winery's new wine was introduced to compete with our most popular wine, which is sold in a distinctive tall, black bottle.  Danville uses a similar bottle.  Thus, it is likely that many customers intending to buy our wine will mistakenly buy theirs instead.
Danville Winery:  Not so.  The two bottles can be readily distinguished:  the label on ours, but not on theirs, is gold colored.
Weaken D: 消费者更多关注瓶子,很少注意商标
(A) Gold is the background color on the label of many of the wines produced by Danville Winery. ---fact, 支持D
(B) When the bottles are viewed side by side, Danville Winery's bottle is perceptibly taller than Mourdet Winery's. ---支持D
(C) Danville Winery, unlike Mourdet Winery, displays its wine's label prominently in advertisements. ---支持D
(D) It is common for occasional purchasers to buy a bottle of wine on the basis of a general impression of the most obvious feature of the bottle. ---correct, 因为排除了其它,但是有一点疑惑,most obvious feature没有阐明到底是什么feature…
(E) Many popular wines are sold in bottles of a standard design. ---无关

5.    32s
Finding of a survey of Systems magazine subscribers: Thirty percent of all merchandise orders placed by subscribers in response to advertisements in the magazine last year were placed by subscribers under age thirty-five.
Finding of a survey of advertisers in Systems magazine: Most of the merchandise orders placed in response to advertisements in Systems last year were placed by people under age thirty-five.
Must be true: 没太明白题目…推测不出来
猜E吧。。。
作者: hanhan1991    时间: 2012-11-14 12:26
1.
1)计时:51s
2)逻辑链:
Premise: The carpet market is tied to the size of population for people buy carpet only once or twice, first in their 20s-30s and the second in their 50s-60s.
Conclusion:Carpet companies can increase their market share only thtough purchasing their competitiors but not through more aggressive marketing.
3)推测:
There are other methods to increase the market share othen than buying their competitors, such as expending their overseas market.In addition, if the number of people in their 20s-30s or in their 50s-60s increase in recent years, the companies can expend their market share through more aggresive marketing.(opposite to the premise)
4)答案分析:选D.
A.irrelevant.
B.Though no new producers enter the carpet markets, the market share of the company still can not increasing.
***if there is no new niche for new brands to fill ,the company can increasing its market share only by mergering.
C.irrelevant.
***profits is not eqeal to market shares.
D.If other producers leave the market, then the producer can attract new customers who buy carpets from their competitors,thereby increasing the market share through a more aggressive marketing.
E.irrelevant.**strethgen

2.
1)计时:39s
2)逻辑链:
Background: Increasing the amount of carbon dioxide absorbed by plant life can help remove excess carbon dioxide in the atmosphere.
Objective:Reduce the amount of carbon dioxide in the atmosphere.
Plan:Establish giant seaweed in the ocean and burn them when they die.
3)推测:
If the carbon dioxide the seaweeds bringing out in the process of burning is the same as that they absorbs in their lifetime, the plan can not achieve its goal.
4)答案分析:选B.
A.Some is not representative.
B.The same as the prediction.
C.Some is not representative.
D.This still can reduce the amount of carbon dioxide in the atmosphere.strenghten.
E.money is irrelevant.

3.
1)计时:59s
2)逻辑链:
Background: The number of Video rentals in 1994 is 10,000 lower than that in 1993.
Explanation: It is due to the opening of Videorama, which sold vedios cheaply.
Premise: The number of the video sold is much less than the total decreasing number of video rentals.
Conclusion: The explanation is unjusted.
3)推测:
Though the number of video sold can not account for the overall drecreasing number of video rentals , people who buy videos can rental it to their frend which  also can lead to the decline in the number of video rentals.
4)答案分析:选E.
A.strengthen.
B.strenthten.
C.irrelevant.
D.irrelevant.
E.the same as prediction.

4.
1)计时:43s
2)逻辑链:
Background: DW's new new wine was put into bottles that is smilar to MW's.
Premise:The gold label of DW on the bottles is different from that of MW.
Conclusion:Costomers will not mistaken these two kinds of wines.
3)推测:
If people distinguish the wine only by its shap of bottle, they will mistakenly buy DW's instead.
4)答案分析:选D.
A.irrelevant.
B.irrelevant.
C.irrelevant.
D.the shape other than the color of the labe is the most obvious features of the bottle. so people buy wine on the base of the shape of the bottle, and mistaken DW's wine for MW's.
E.irrelevant.

5.
1)计时:50s
2)逻辑链:
Fact1: 30% of the all merchandise orders placed by subscribers in reponse to ad in the magazine were placed by subscribers under 35.
Fact2:Most of the merchandise orders placed by people in reponse to ad were placed by people under 35.
3)推测:
Most people  perchase merchandise orders in response to ad are not the subscribers of the magzine.
4)答案分析:选E.
A.wrong. it shoule be less not more.(subscribers中两个变量age/order,用表格法即可求)
B.not mention now.
C.wrong.the percentage is less than 30%.
D.the dollar is not mentioned.
E.the same as predicion.
作者: CD用户825193    时间: 2012-11-14 20:06
标题: Daily CR-3_2012-11-114
1. 42" (Weak) [晕,又看一遍:40"]
Background: There's little room of carpet market, which is tied to the size of population. People only buy carpet once or twice in their whole lief.
Premise: As people get old.
Conclusion: Carpet company will get more profit only through purchasing competitor (?), not aggressive marketing.
Prephrase:
As the first purchased customers getting old, there'll also be some new customers growing to the age of 20s or 30s. - Aggressive marketing will still work.
Marketing overseas in a new market?
(WRONG GUESS)
Didn't understand the key point of this question: market share.
Answers:
(A) "floor covering" is IRRELEVANT
(B) "no remaining niche for a new brand" IRRELEVANT - SUPPORT
This is an Opposite answer that strengthens the argument.
If there are no remaining niches to fill, then there is no way to expand other than
to purchase a competitor.

(C) "2/3 emerged companies declined in profit" explain the market is hard to get in, but doesn't doubt the conclusion. (Didn't understand Answer-C: MERGER-NOT imerge)
Answer choice (C) simply suggests that when companies purchase
their competitors the endeavor is often financially unsuccessful. Essentially,
answer choice (C) fails to prove that purchasing competitors is unnecessary to
gain market share.

(D) CORRECT
"the dominant company cut price through lower cost so that others leave the market" doesn't doubt the conclusion.
* Cutting down the cost -> Lower prices -> Competitors leaving -> Bigger market share
(E) CORRECT.If consumers increasingly follow the new patterns and styles.
"consumers are increasingly resistant (=reluctant) to new patterns and styles"

[好几处都理解错了,一塌糊涂。。。]

2. 26" (Weak)
Background: Carbon dioxide of the atmasphere keeps increasing.
Premise: One seaweed can reduce the carbon dioxide.
Conclusion: Growing large seaweed on the ocean, then burn it for fule.
Prephrase: Burning seaweed will produce more (OR COMPARABLE) carbon dioxide into the atmasphere than it reduces.
Answers:
(A) Some areas of the oceans don't support the seaweed farm, but there're still other areas do.
(B) CORRECT.
(C) If people wanna use the new fuel is irrelevant.
(D) No seaweed farm mentioned, irrelevant.
(E) Nothing about the carbon dioxide removed, irrelevant.

3. 42", 1'20"+ (Weak)
Premise: 10,000 video rentals declined in 1994 than 1993.
Premise: Videorama is the only shop renting videos, also selling them cheaply.
               Videorama opend in Feb, 1994.
               Videorama only sold 4,000 videos in 1994
Conclusion: SVideorama is not the reason 10,000 video rentals declined.
Prephrase: 10,000 rentals contained same videos.
Answers:
(A) Irrelevant
(B) Irrelevant
(C) Irrelevant
(D) Irrelevant
(E) CORRECT: people buy video -> lend to friends -> friends stop renting the movie ->less rentals

4. 29" (Weak)
Premise: Competitor's new wine bottle is similar to ours
                 The lable on our bottle is golden, but not the competitor's.
Conclusion: Customers will not mistakenly buy their wine.
Prephrase: Customers don't realize the diffrence of the lables.
Answers:
(A) Support.
(B) Golden lable is not mentioned. Irrelevant
(C) Support
(D) CORRECT
(E) Irrelevant

5. " (Conclusion)
Premise: Orders by 35- yrs subscribers/ Orders by all subscribers = 30%
                 Orders by 35- yrs / Orders by all in response to the magzine ad = most
Conclusion: Lots of 35- yrs buyers r not subscribers
Answers:
(A)
(B)
(C)
(D)
(E) CORRECT
作者: fpd    时间: 2012-11-21 23:04
2.background:   scientists decide to remove excess carbon dioxide from the atmosphere by increasing the amount...

fact:  establish giant floating seaweed farms in the oceans and when seaweed die, they will be burned for fuel.

speculation:  what if burn thses seaweed cause more carbon dioxide(典型的, 题目作者没有考虑完全所有的可能性)
A。没关系,这题讨论的是,二氧化碳和seaweed 的关系。
B。对
C。别人reluctant 和premise 和conclusion 没关系。
D。这个更没关系。这个攻击的是backgroud。
E。和降低二氧化碳没关系

3.conclusion: vedio rental decline 10.000in 1994
premise: an openging of a vedio market, not only rent but alos sell cheaply.
attack:  the market only sell 4.000 videos  in 1994.
speculation:  the decline is also caused by other facts of the opening of V,not only depend on the selling.

a. 这是支持的, 租出去更多的video, 那么rental 应该是增加的。
b。 这也是支持的, 开了更多的租碟点。那么rental 应该增加的。
c。这也是增加的。 租碟在特定week 打折,那么, rental 也是会增加的。
D。无关。
E有关。  经常借给别人,那么,虽然卖出去了那么多,但是,组的,会成倍减少。

4,  premise:DW 's new wine use the same bottle with us.
    conclusion: customers will buy by mistake.
   attackW:we have golden colored label.
 speculation:  customer can't distinguish the label?  MW also have agodlen label?
A right.(错误了!  other types of wines 是无关的!)
B 没关系。
C。没关, 或者,支持了,DW的观点。
D。也是支持了DW的观点。
E。没关。


5.(这个题,题目灭看懂)一开始选A,后来选C,再后来,选D。, key words:  subscriber   under age 35
 把这个题目翻译一下,就明白了。  
 在一个magazine 的subscribers 的调查中,得出的:    通过杂志发订单的人35%是低于35岁得。
 广告商的调查:    大部分订单来自 低于35岁的人。


BEAE
作者: kudoucliff    时间: 2012-12-16 13:13
3. There is relatively little room for growth in the
overall carpet market, which is tied to the size of the
population. Most who purchase carpet do so only
once or twice, first in their twenties or thirties, and
then perhaps again in their fifties or sixties. Thus as
the population ages, companies producing carpet
will be able to gain market share in the carpet market
only through purchasing competitors, and not
through more aggressive marketing.
Which one of the following, if true, casts the most
doubt on the conclusion above?
(A) Most of the major carpet producers market
other floor coverings as well.
Whether the producers market other products has no bearing on the carpet market.

(B) Most established carpet producers market
several different brand names and varieties,
and there is no remaining niche in the
market for new brands to fill.
The truth doesn’t weaken the conclusion got by the author; no remaining niche market can leads to both merging and fierce competition.

(C) Two of the three mergers in the industry’s last
ten years led to a decline in profits and
revenues for the newly merged companies.
The author’s conclusion focus on the market share got by merging rather than revenues.

(D) Price reductions, achieved by cost-cutting in
production, by some of the dominant firms
in the carpet market are causing other
producers to leave the market altogether.
This truth weakens the author’s conclusion soundly; since price competition leads to some companies’ failure, the author’s claim that companies cannot gain market share through aggressive market is invalid.

(E) The carpet market is unlike most markets in
that consumers are becoming increasingly
resistant to new patterns and styles.
Whether customers are resistant to new patterns and styles is irrelevant with the author’s conclusion.

逻辑链:
Truth 1: Growth room of carpet market is little.
Truth 2: The frequency of purchase is low, as the product is enduring.
Supposition: Companies will be only able to gain market share by merging competitors rather than by marketing more aggressively.
Weaken point: There are companies that failed in the market because of aggressive competition.

Aim: To remove excess carbon dioxide in air.
Proposed plan: To establish giant floating seaweed farms in the oceans. When they died, can be disposed of by being burned for fuel.
Weaken point: When they were burned, they release carbon dioxide as much as they absorb.

Truth: Video rental outlet was 10000 fewer in 1994 than that in 1993.
J said: For video rental shops also sell them cheaply.
B said: But the number sold is only 4000, while the number of declined rental is 10000.
Weaken B: People buy videos frequently borrow them to their friends.

Truth: DW’s new wine is similar in appearance with MW’s wine.
MW: Customers will mistake DW’s wine for MW’s wine.
DW: The label on ours is different from that of theirs.
Weaken DW: Customers purchase a bottle of wine on the basis of a general impression.

Finding 1: 30% of orders placed by subscribers in response to AD in the magazine last year were placed by subscribers under 35.
Finding 2: Most of orders placed in response to AD in the magazine last year were placed by people under 35.
Support point: Most subscribers under 35 place orders in response to AD in the magazine are not subscribers of the magazine.
第五题我做了整整四分钟才整明白啊!!!简直不能忍!!
作者: annieliu830    时间: 2012-12-23 00:34
2012年12月23日
1.
1)逻辑链
原因:人口数量有限,而每个人又只买一到两次地毯
结论:生产地毯的企业要想扩大市场,只有通过购并竞争者,而不是市场策略
2)推测
削弱
通过更激进的市场策略,可以使未有购买倾向的顾客购买自己家的地毯
3)选项分析
A无关
B加强
C无关,利润与市场占有率无关
D正确,没有通过购并竞争者,却也使得市场份额有机会增加
E无关

2.
1)逻辑链
目的:通过植物吸收使大气中的二氧化碳减少
措施:在海上种植漂流植物,在植物死了之后,再燃烧了做燃料
2)推测
削弱
植物死了之后燃烧会释放出更多的二氧化碳
3)选项分析
A只说在南半球采取此措施可能不可行,但若在北半球采取此措施是可行的,则还是可以吸收很多二氧化碳,还是有用的
B正确。
C与是否换燃料无关
D加强,说明还是有用的
E与之后怎么处理farming seaweed无关,只与seaweed吸收了二氧化碳有关

3.
1)逻辑链
Jennifer:现象:1994年C的影碟出租比1993年少了1万
解释:是因为1994年V的开张,还有卖碟
Brad:
反驳解释:少租了10000张,却只多卖了4000张叠,说明还有其他原因
2)推测
Weaken brad 加强jennifer
卖了碟还会借给其他人看,所以导致少租了这么多张
3)选项分析
A无关,因为题目中说的是,租的碟的总数(即也包括V租的)少于去年
B无关
C与租的价格高低无关
D那只能说明买的数量增加,与租的数量无关
E正确

4.
1)逻辑链
M W:原因:DW的新产品与我们最流行的啤酒有竞争,他们都是用的类似的包装
结论:有的可能想买我们产品的,因为包装类似而买了对方的
DW:weaken结论:我们不一样,标签不一样,我们的是金色的
2)推测
削弱
不一样的地方不显眼,不容易被发现,
3)选项分析
A无关,只说了DM的
B加强
C加强
D正确
E加强

5
1)逻辑链
SM订阅者的调查结果:杂志订阅者中30%因为广告而下订单购买的是35岁一下的订阅的
SM广告者的调查结果:大多数因为广告而订阅的都是35岁以下的
2)推测
两者都真实
大多数杂志订阅者都是35岁以上的
3)选项分析
A与题目中的条件二相矛盾
B今年与    去年比较,无关
C与题目中的条件二相矛盾
D只能由条件一推出是正确的,却无法得知是不是两者同时正确
E正确
作者: zxppx    时间: 2013-1-8 19:36


3. There is relatively little room for growth in the
overall carpet market, which is tied to the size of the
population. Most who purchase carpet do so only
once or twice, first in their twenties or thirties, and
then perhaps again in their fifties or sixties. Thus as
the population ages,
companies producing carpet
will be able to gain market share in the carpet market
only through purchasing competitors
, and not
through more aggressive marketing.
Which one of the following, if true, casts the most
doubt on the conclusion above?
--BIBLE WEAKEN NO.3

(A) Most of the major carpet producers market
other floor coverings as well.

There is no relationship with other floor coverings.
(B) Most established carpet producers market
several different brand names and varieties,
and there is no remaining niche in the
market for new brands to fill.

It strengthens the conclusion, instead of weakening.
(C) Two of the three mergers in the industry’s last
ten years led to a decline in profits and
revenues for the newly merged companies.

It tells that merging perhaps is bad. However, we should notice that the phenomenon is not sufficient to represent the whole situation.
(D) Price reductions, achieved by cost-cutting in
production, by some of the dominant firms
in the carpet market are causing other
producers to leave the market altogether.

It strengthens the conclusion that the carpet market needs merging, rather than more aggressive marketing.

It reveals that if firms find ways to cut cost in production, they will benefit from the action, causing other producers to leave the market altogether.
(E) The carpet market is unlike most markets in
that
consumers are becoming increasingly
resistant to new patterns and styles.

If ‘in that’ is referred to most markets, the answer means that the consumers in the carpet market tend to like more aggressive markets, rather than merging. So it is the best answer.

It means that the consumers in the carpet market are becoming increasingly resistant to new patterns and styles. So we need no more aggressive marketing. It strengthens the consluion.

2.
Scientists are discussing ways to remove excess carbon dioxide from the atmosphere by increasing the amount that is absorbed by plant life. One plan to accomplish this is to establish giant floating seaweed farms in the oceans. When the seaweed plants die, they will be disposed of by being burned for fuel.

Which of the following, if true, would indicate the most serious weakness in the plan above?
-- PREP07 CR1 NO.9


(A) Some areas of ocean in the Southern Hemisphere do not contain sufficient nutrients to support large seaweed farms.

Although the argument seems to weaken the plan, the phenomenon is unrepresentative to reveal the whole condition, considering that other areas of ocean may contain sufficient nutrients.
(B) When a seaweed plant is burned, it releases an amount of carbon dioxide comparable to the amount it has absorbed in its lifetime.

The purpose of the plan is to remove excess carbon dioxide. If the amount of carbon dioxide cannot be reduced through the plan, there is no reason to advocate the plan. So it weakens the argument.
(C) Even if seaweed farms prove effective, some people will be reluctant to switch to this new fuel.

We need not consider that whether some people would be reluctant to conduct the plan.
(D) Each year about seven billion tons of carbon dioxide are released into the atmosphere but only about five billion tons are absorbed by plant life.

It seems to relate with the first sentence in the argument, but it cannot decide the availability of the plan, because the argument talks about the ‘excess carbon dioxide’.
(E) Seaweed farms would make more money by farming seaweed to sell as nutritional supplements than by farming seaweed to sell as fuel.
There is no relationship with money.



3.

Jennifer: Video rental outlets in Centerville together handled 10,000 fewer video rentals in 1994 than in 1993. The decline in rentals was probably due almost entirely to the February 1994 opening of Videorama, the first and only video rental outlet in the area that, in addition to renting videos, also sold them cheaply.

Brad: There must be another explanation: as you yourself said, the decline was on the order of 10,000 rentals. Yet Videorama sold only 4,000 videos in 1994.


Which of the following, if true, would most seriously weaken the force of the objection that Brad presents to Jennifer's explanation?
-- PREP07 CR1 NO.10


(A)In 1994 Videorama rented out more videos than it sold.

Videorama is unrepresentative to the video rental outlets in Centerville. Besides, the condition in Videorama is opposite to the whole trend.
(B) In 1994 two new outlets that rent but that do not sell videos opened in Centerville.

The two new outlets cannot help us to understand more about the real reason of the decline.
(C) Most of the video rental outlets in Centerville rent videos at a discount on certain nights of the week.

If the fact is true, the revenue in rentals should rise, because the discount stimulates the demand.
(D) People often buy videos of movies that they have previously seen in a theater.

If the fact is true, the revenue in rentals should rise.
(E) People who own videos frequently loan them to their friends.

It most seriously weakens the objection provided by Brad, because if the people who own videos frequently loan them to other people, even 4,000 is smaller than 10,000, people who need to rent videos from rental outlets can just borrow them from their friends, thus weakening the objection.



4.
Mourdet Winery: Danville Winery's new wine was introduced to compete with our most popular wine, which is sold in a distinctive tall, black bottle. Danville uses a similar bottle. Thus, it is likely that many customers intending to buy our wine will mistakenly buy theirs instead.


Danville Winery: Not so. The two bottles can be readily distinguished: the label on ours, but not on theirs, is gold colored.


Which of the following, if true, most undermines Danville Winery's response?
-- PREP07 CR1 NO.11

(A) Gold is the background color on the label of many of the wines produced by Danville Winery.

It cannot weaken Danville Winery’s response.
(B) When the bottles are viewed side by side, Danville Winery's bottle is perceptibly taller than Mourdet Winery's.

It strengthens DW’s response, since we can see the distinction between them.
(C) Danville Winery, unlike Mourdet Winery, displays its wine's label prominently in advertisements.

It cannot weaken DW’s response, because displaying label prominently in advertisements has no relationship with similarity of the bottle.
(D) It is common for occasional purchasers to buy a bottle of wine on the basis of a general impression of the most obvious feature of the bottle.

Since consumers will have the most impression with the bottle, it weakens DW’s response. So it is the best answer.
(E) Many popular wines are sold in bottles of a standard design.
It weakens MW’s reason, rather than DW’s response.



5.
Finding of a survey of Systems magazine subscribers: Thirty percent of all merchandise orders placed by subscribers in response to advertisements in the magazine last year were placed by subscribers under age thirty-five.


Finding of a survey of advertisers in Systems magazine: Most of the merchandise orders placed in response to advertisements in Systems last year were placed by people under age thirty-five.


For both of the findings to be accurate, which of the following must be true?
-- PREP07 CR1 NO.12

(A) More subscribers to Systems who have never ordered merchandise in response to advertisements in the magazine are age thirty-five or over than are under age thirty-five.

We know nothing about subscribers thirty-five or older.
(B) Among subscribers to Systems, the proportion who are under age thirty-five was considerably lower last year than it is now.

There is no comparison with subscribers between last year and this year.
(C) Most merchandise orders placed in response to advertisements in Systems last year were placed by Systems subscribers over age thirty-five.

The second sentence tells us that most merchandise orders were placed by people under age 35.
(D) Last year, the average dollar amount of merchandise orders placed was less for subscribers under age thirty-five than for those age thirty-five or over.

We know nothing about the average dollar amount of merchandise orders.
(E) Last year many people who placed orders for merchandise in response to advertisements in Systems were not subscribers to the magazine.

It seems to irrelevant to the argument, but if you consider more deeply, you’ll find that it is the best answer. Because 30% of the orders placed by subscribers were under 35, and most of the orders placed by people were under 35, there are more people who placed the orders were not subscribers.

作者: pennyz    时间: 2013-2-5 23:33
46sweaken
p:the number of selling is restricted by population
c:the growth can only attribute to the competitor's share
rephrase:there might be other way to stimulate consumers to buy more
D
48S
B:When the plant die ,we burn them
aim:reduce the amount of carbon dioxide through increasing the amount taken in by plants
way:grow more plant in ocean
the dispose of the die plant may cause way too much carbon dioxide
B
57SWEAKEN
NO experience in the dialogue type
J:
result:the rent fall
reason:new store rent+sell cheap
B:rent fall>copy sell
rephrase:people who buy the copies might share them with the potential comstomers who
are willing to rent
E
37S
the new brand wine use similar bottle as our brand does
defeat:the name of ours is golden,which is different
rephrase:the golden color may not be so explicit that customers may omit
D

45S
The subscriber of age under 35 is about 30%
the order of ads mostly are age under 35
conclusion:the people under 35 mainly read this  magizine to shop(little wierd?)
A?
作者: 一粒黄豆。    时间: 2013-3-7 18:52
3/7
38'
premise:sales of carpet are limited because of population
conclusion:company could gain market share only through purchasing competitors, and not through developing market.
prephrase: what if population increases?
A irrelevant
B irrelevant
C irrelevant
D irrelevant

E correct


33'
premise:seaweed--> absorb co2; dead seaweed can be burned
conlcusion:grow seaweed--> remove excess co2
prephrase:co2 released by process of burning seaweed may be greater than co2 absorbed by seaweed.
B



第一遍1'04'
J:V open--> rentals decrease
B: another explanation: V 4000   rentals 10000
prephrase: this figure doesn't have point,  we should compare rentals in 1994 to that before, V was just opened ,it is normal to have 4000
看了选项以后才明白自己理解错误
J: V open--> rentals decrease by 10,000
B: V sold 4000, which is smaller than 10,000
prephrase:V rent 6000 maybe
A correct



48'
MW: bottles are similar, customer will buy DW's by mistake
DW: our lables are good colored
prephrase: customers don't know whose are good colored, they still could not distinguish between two brands
D



第一遍1'
1 30% subscriber responsed to ad are under 35
2  most ....under 35
prephrase:percentages of each other groups are less than 30%, so the 30% is the largest one
理解错误= =
S--30%---<35
AD--most--<35
prephrase:<35----AD
这个题有点难啊!!

还是中文理一遍吧!
sys:sys杂志的订购者中,因为广告而下商品订单的人中 30%小于35岁
ad:来自sys杂志的订单,大多数都是小于35岁的人买的

--->差距在于是不是杂志的订购者,说明很多小于35岁的非sys杂志订购者的人 下了商品订单

E
作者: okplokpl0714    时间: 2013-3-28 08:38
我去。。。刚打好的一堆回帖给误删了!!!气人!!!
精练:
Answer:E
错误原因:
没有理解D选项的真谛。。。我知道了,必要条件有两个,攻击其中一个也是答案啊,有收获!!
还有。。不知道in that到底修饰啥的!!这个语法知识点我真的不知道啊!!求指导!!
逻辑链:
2.猜测:燃烧seawood 产生CO2也不少,选B
3.猜测:他因,rideo可以不用多买,选E
4.猜测:DW的lable不引人注目,选D
5.猜测:advertisers>subscribes,选E
作者: okplokpl0714    时间: 2013-3-28 09:02
in that是因为的意思。。。我勒个去。。这个初中英语知识点我竟然忘掉了。。!!TAT!!
作者: ChangTienYi    时间: 2013-3-29 20:06
我的答案是1D 2B 3E 4D 5A,求更正
作者: none123    时间: 2013-5-4 15:33
1: 52‘ premise:growth of the carpet industry is very difficult and carpet company only do so twice during their lifetime.
conclusion:so a carpet company can grow only by purchasing competitors
pre: decrease the price of the produts can attract consumers.
answers:
A irrelevant : passage is about ways to increase the share in the carpet industry..
B strengthen the argument
Cirrelevant
D right .because by the marketing strategy of  price deduction can increase the share in this industry.
E irrelavant .it does nothing with the aggressive marketing.
2: 24' premise: the plant can absorb the excess carbon dioxide  and the plant can be burned by the fuel.
conclusion:the  plant should be used to deal with the excess carbon dioxide.
pre:the burning process of this plant makes a significant amount of carbon dioxide.
answer:
CE irrelevant : C talks about the fuel,and E talks about the irrelevant conparison.
B right.because the fact makes it impossible for this solution to achieve this goal
A i know it is not right,but i don't know why it is wrong.maybe the nutrition can be supplied by manual work.
3: 44'premise :the rental outlet of the video in most of the shops decreased 1000 last year.Videorama is the first shop to sell and rent at the same time. the selling revenue is 4000.
conclusion:the rental decrease has notjing to do with the Videorama.
pre:maybe the people buy the video can copy it and sell the copy again.
answer:E right.becuase to weak the argument,we must say the opening of Videorama is the reason.
4: 48' A and B companies have the similar bottle of the wine. A think consumers intend to buy their wine buy B's mistakely.
conclusion:bacuase the color of label is different,so it is not ture.
pre:label is too small that consumers don't pay attention to the colour of label.
answer:right because the height is the thing that consumers first see,so the label is been ignored.
C irrelecant.it isn't talking about the advertisemnt
5: 38'
Eis right.
by the comparison of the 30% subscribers and  most of the oders.
作者: Mint静默    时间: 2013-7-2 01:13
1.
1)  计时:36s
2) Premise:
Most who purchase carpet do so only once or twice, first in their twenties or thirties, and then perhaps again in their fifties or sixties.
Conclusion:
As the population ages, companies producing carpet will be able to gain market share in the carpet market only through purchasing competitors, and not through more aggressive marketing.
3) 推测(weaken)
说实话,没什么思路。。。
4) 选项分析:
(A) Most of the major carpet producers market
other floor coverings as well.
无关,other floor coverings
(B) Most established carpet producers market
several different brand names and varieties,
and there is no remaining niche in the
market for new brands to fill.
无关
(C) Two of the three mergers in the industry’s last
ten years led to a decline in profits and
revenues for the newly merged companies.
无关,Decline in profits and revenues 不等于 decline in market share
(D) Price reductions, achieved by cost-cutting in
production, by some of the dominant firms
in the carpet market are causing other
producers to leave the market altogether.
Correct, 说明marketing可以抢占市场份额
(E) The carpet market is unlike most markets in
that consumers are becoming increasingly
resistant to new patterns and styles.
无关,有support之嫌


2.
1)  计时:24s
2) Premise:
When the seaweed plants die, they will be disposed of by being burned for fuel.
Plan:
Accomplish this is to establish giant floating seaweed farms in the oceans.
Goal:
To remove excess carbon dioxide from the atmosphere by increasing the amount that is absorbed by plant life.
3) 推测(weaken)
Seawed plants 燃烧的时候产生更多的CO2
4) 选项分析:
(A) Some areas of ocean in the Southern Hemisphere do not contain sufficient nutrients to support large seaweed farms.
无关
(B) When a seaweed plant is burned, it releases an amount of carbon dioxide comparable to the amount it has absorbed in its lifetime.
Correct
(C) Even if seaweed farms prove effective, some people will be reluctant to switch to this new fuel.
无关
(D) Each year about seven billion tons of carbon dioxide are released into the atmosphere but only about five billion tons are absorbed by plant life.
不能说明不effective
(E) Seaweed farms would make more money by farming seaweed to sell as nutritional supplements than by farming seaweed to sell as fuel.
与money无关


3.
1)  计时:27 s
2) J:
The decline in rentals was probably due almost entirely to the February 1994 opening of Videorama that in addition to renting videos, also sold them cheaply.
B:
The decline was on the order of 10,000 rentals. Yet Videorama sold only 4,000 videos in 1994.
3) 推测(weaken B)
V家的sell带动了V家的租碟业务,从而也影响了总的decline
4) 选项分析:
(A) In 1994 Videorama rented out more videos than it sold.
无关
(B) In 1994 two new outlets that rent but that do not sell videos opened in Centerville.
irrelevant
(C) Most of the video rental outlets in Centerville rent videos at a discount on certain nights of the week.
irrelevant
(D) People often buy videos of movies that they have previously seen in a theater.
Weaken J, not B
(E) People who own videos frequently loan them to their friends.
Correct


4.
1)  计时:24s
2) 推测(weaken Danville)
一些顾客可能没有注意到label
3) 选项分析:
(A) Gold is the background color on the label of many of the wines produced by Danville Winery.
无关
(B) When the bottles are viewed side by side, Danville Winery's bottle is perceptibly taller than Mourdet Winery's.
Support,not weaken
(C) Danville Winery, unlike Mourdet Winery, displays its wine's label prominently in advertisements.
Support, not weaken
(D) It is common for occasional purchasers to buy a bottle of wine on the basis of a general impression of the most obvious feature of the bottle.
可能不会注意到label
(E) Many popular wines are sold in bottles of a standard design.
无关



5.
1)  计时:1min42s
3) 推测
题中信息说明Systems magazine不符寻常?
4) 选项分析:
(A) More subscribers to Systems who have never ordered merchandise in response to advertisements in the magazine are age thirty-five or over than are under age thirty-five.
who have never ordered merchandise in response to advertisements不在考虑范围内
(B) Among subscribers to Systems, the proportion who are under age thirty-five was considerably lower last year than it is now.

(C) Most merchandise orders placed in response to advertisements in Systems last year were placed by Systems subscribers over age thirty-five.
和正确选项E相反
(D) Last year, the average dollar amount of merchandise orders placed was less for subscribers under age thirty-five than for those age thirty-five or over.
不一定,merchandise orders包括in response to advertisements和not in response to advertisements.
(E) Last year many people who placed orders for merchandise in response to advertisements in Systems were not subscribers to the magazine.
Correct

作者: 黄小鹤    时间: 2013-7-19 22:01
1.B: Carpet has little room for growth because it relates to the size of the population.
P: People at their 20s and 50s are likely to buy carpets.
C: Carpet companies could gain market share by purchasing competitors but not by aggressive market strategy.
[weaken]
推测:
选项分析:(A) Most of the major carpet producers market other floor coverings as well.      

(B) Most established carpet producers market several different brand names and varieties, and there is no remaining niche in the market for new brands to fill.

(C) Two of the three mergers in the industry’s last ten years led to a decline in profits and revenues for the newly merged companies.      Right. 我个人是把它看成“方案推理”题型,而该选项涉及方案。

(D) Price reductions, achieved by cost-cutting in production, by some of the dominant firms in the carpet market are causing other producers to leave the market altogether.

(E) The carpet market is unlike most markets in that consumers are becoming increasingly resistant to new patterns and styles.        

2. B  3. E   4. D   5. ?不懂。。

作者: 夏日之浅浅    时间: 2013-8-4 00:27
求第五题翻译。。。。
作者: qinqinyu    时间: 2013-8-6 17:58
请问后面四题的正确答案在哪呢?

作者: Elisha728    时间: 2013-8-26 07:48
CBADE
Time:8'15
作者: Elisha728    时间: 2013-8-27 07:14
why the third question is "E"?

Could any DN help explain?
作者: m1nt    时间: 2013-8-31 17:06
1.        35’
Pre: carpet market, which is tied to the size of the population
Con:        gain market share in the carpet market only through purchasing competitors
Weaken:
D right
ABCE 无关

2.        15’
Pre: When the seaweed plants die, they will be disposed of by being burned for fuel.
Con: Seaweed will remove DC
Weaken: Seaweed die will release more DC
B right

3.        30’
J: VR sell cheap, 10000 fewer rentals
B: VR sold only 4000
Weaken B: people bought at VR then lend to friends

4.        20’
MW: D uses similar bottle- customers intending to buy ours will buy theirs
D: bottles are different, D‘s with gold label
Weaken: customers won’t notice the label
ABC Support
D right
E 无关

5.        30’
Fact1: 30% orders by subscribers were placed by under 35
Fact2: Most orders were placed by under 35
Must be true: many are not subscribers

作者: meckyona    时间: 2013-9-5 22:19
【精练】
3. There is relatively little room for growth in the
overall carpet market, which is tied to the size of the
population. Most who purchase carpet do so only
once or twice, first in their twenties or thirties, and
then perhaps again in their fifties or sixties. Thus as
the population ages, companies producing carpet
will be able to gain market share in the carpet market
only through purchasing competitors, and not
through more aggressive marketing.
Which one of the following, if true, casts the most
doubt on the conclusion above?
--BIBLE WEAKEN NO.3
28’
B: there’s little room for growth in carpet market
P:most people buy carpets only for 2 times, in 20~30 and 50~60
C: companies only can gain share through purchasing competitors
问削弱

A:focus on carpet
B: irrelevant
C:irrelevant
D:削弱了原文观点,right
E:irrelevant

2        23s
B:remove carbon dioxide
C: plant floating seaweed farms in oceans, burned them to fuel when thy died
问削弱  也许烧海草还是会产生carbon 或者生长过程产生

A:相关削弱
B: 强势削弱~right
C:irrelevant
D: irrelevant
E 无关

3        29s
B: j:video rental 10000 fewer in 1994 than 1993, maybe due to videoroma that sold them cheaply
b only sold 4000, must be another explanation
问削弱B

A:不削弱
B 加强
C 无关
D right 不知道做这题时怎么搞的= =答案是E
E 无关

26s
B: mw’s wine may mistaken by customers as aw’s
P: dw’s wine has golden label on blake bottles
问削弱dw
可能顾客不注重label, 可能金色和黑色混淆

A:加强
B 加强
C 加强
D right
E 无关

48s 没太懂= =
Subscriber 订户

排除B C D
A吗。。。 E

作者: lyrsilvia    时间: 2013-9-13 16:58
1. 30'
BG: There is relatively little room for growth in the overall carpet market, which is tied to the size of the
population.
P: Most who purchase carpet do so only once or twice, first in their twenties or thirties, and then perhaps again in their fifties or sixties.
C: companies producing carpet will be able to gain market share in the carpet market only through purchasing competitors, and not through more aggressive marketing
Doubt: purchase competitors cannot guarantee to gain market share
答案:C   D
A) Most of the major carpet producers market other floor coverings as well.-----what producers produce is irrelevant
(B) Most established carpet producers market several different brand names and varieties, and there is no remaining niche in the market for new brands to fill.-------new brands is not discussed
(C) Two of the three mergers in the industry’s last ten years led to a decline in profits and revenues for the newly merged companies.-------seems correct to illustrate the disadvantage of purchasing.
(D) Price reductions, achieved by cost-cutting in production, by some of the dominant firms in the carpet market are causing other producers to leave the market altogether.--------price reductions strategy is irrelevant
(E) The carpet market is unlike most markets in that consumers are becoming increasingly resistant to new patterns and styles.-----------new patterns and styles are irrelevant
============================================================================================================
错误分析:C项虽然说了purchasing disadvantages,but it will not weaken the plan to increase market share.
                 D is correct because it illustrate that there may be some leaving in the market because of the price reduction. These leaving will  give room for growth in the overall carpet marke, means more aggressive marketing will increase companies market share. doubt that the purchase is the only method.又是看了D的头没有看D的尾巴,粗心粗心啊
===========================================================================================================
2. 30'
BG: Scientists are discussing ways to remove excess carbon dioxide from the atmosphere by increasing the amount that is absorbed by plant life.
P: establish giant floating seaweed farms in the oceans; When the seaweed plants die, they will be disposed of by being burned for fuel.
C:  remove excess carbon dioxide from the atmosphere
Weaken: the plan cannot remove the carbon dioxide
答案:B
(A) Some areas of ocean in the Southern Hemisphere do not contain sufficient nutrients to support large seaweed farms.---------some areas is not all areas, the plan may still be effective
(B) When a seaweed plant is burned, it releases an amount of carbon dioxide comparable to the amount it has absorbed in its lifetime.--------means absorb=release, no remove. correct
(C) Even if seaweed farms prove effective, some people will be reluctant to switch to this new fuel.-----------what some people do may not affect the whole situation("some" is bad here)
(D) Each year about seven billion tons of carbon dioxide are released into the atmosphere but only about five billion tons are absorbed by plant life.-----------means large increase the plant is needed, support not weaken
(E) Seaweed farms would make more money by farming seaweed to sell as nutritional supplements than by farming seaweed to sell as fuel.-------------make more money is not the subject we discuss

3. 30'
J: 10000 fewer rental is due to the opening of V
B: V just sold 4000, the remaining 6000 is not V's responsibility
Weaken B: 4000 sold will lead to 10000 fewer rentals
答案:E
(A) In 1994 Videorama rented out more videos than it sold.------what V did is irrelevant
(B) In 1994 two new outlets that rent but that do not sell videos opened in Centerville.-------new outlets may be responsible for the missing 6000 rental, support B, not weaken
(C) Most of the video rental outlets in Centerville rent videos at a discount on certain nights of the week.-------how they rent is irrelevant
(D) People often buy videos of movies that they have previously seen in a theater.-------people buy what is irrelevant
(E) People who own videos frequently loan them to their friends.------means buying causes less rental

4. 25'
MW: DW uses similar bottle, customers intending to buy MW will mistakenly buy DW
DW: although similar bottle, but can be distinguished by different label
Weaken DW: different label is not enough for distinguished
答案:D
(A) Gold is the background color on the label of many of the wines produced by Danville Winery.------what detail about the DW's other wine is not relevant
(B) When the bottles are viewed side by side, Danville Winery's bottle is perceptibly taller than Mourdet Winery's.------means there is different, thus consumers can distinguish, support DW
(C) Danville Winery, unlike Mourdet Winery, displays its wine's label prominently in advertisements.------also support DW
(D) It is common for occasional purchasers to buy a bottle of wine on the basis of a general impression of the most obvious feature of the bottle.--------general impression means the slight label different is not enough
(E) Many popular wines are sold in bottles of a standard design.------------also support the DW

5. 30'
F1: 30% of the subscribers who order are under 35
F2: most of the orders are placed by people under 35
MUST BE TRUE: something related to propotion
答案:
(A) More subscribers to Systems who have never ordered merchandise in response to advertisements in the magazine are age thirty-five or over than are under age thirty-five.-----------"more" 没有比较对象
(B) Among subscribers to Systems, the proportion who are under age thirty-five was considerably lower last year than it is now.---------no information about last year's situation
(C) Most merchandise orders placed in response to advertisements in Systems last year were placed by Systems subscribers over age thirty-five.--------F1 states that 30% of S subscribers are under age 35, thus if most merchandise orders placed were placed by S subscribers, the ratio of people under 35 may equal or bigger than 30%, thus it is imprudent for the F2 that most orders are under 35
(D) Last year, the average dollar amount of merchandise orders placed was less for subscribers under age thirty-five than for those age thirty-five or over.---------the average dollar amount is not discussed
(E) Last year many people who placed orders for merchandise in response to advertisements in Systems were not subscribers to the magazine.-----------correct, many people are not subscribers, means many people are under 35, the ratio of these under 35 people is higher in the whole group. thus, only 30% of the subscribers are under 35, but the most people are under 35 is justified
作者: suiqiji206    时间: 2013-9-15 21:40
2m2s
P: c market share = population size=stable
C: only through purchasing competitors can c producers gain market share
Dis correct

1m20s
P:increase CO2 absorbed by plants in order to ruduce CO2
  Plan:establish seaweed farms in the ocean, died seaweed=fuel
C:plan to accomplish the P1
B is correct

1m57s
P: video rentals declind 10,000, Videorama rent and sell vedios
C:the decline is due to the opening of Videorama
C:Videorama  sold only 4,000 vedios
A is correct

1m50s
P:MM and DW produces wines with similar bottles
C:customers won’t buy wines mistakenly due to DW’s golden lable

Eis correct

2m24s
P:30% of all merchandise orders placed by subscribers are under35
  Most of all merchandise orders placed by people are under 35

Eis correct




欢迎光临 ChaseDream (https://forum.chasedream.com/) Powered by Discuz! X3.3